Revision 1: Units 1 and 2

Revision 1: Units 1 and 2 Practice 1 Exercise1. Complete the dialogues with phrases from the box. Nothing much. Cheers. Not at all. How are you? Sl...
Author: Jane May
95 downloads 0 Views 3MB Size
Revision 1: Units 1 and 2 Practice 1 Exercise1. Complete the dialogues with phrases from the box. Nothing much.

Cheers.

Not at all. How are you?

Sleep well.

Excuse me.

Thanks.

Don‟t mention it. Good morning.

Thank you.

Same to you.

Bye for now.

Hi Jan!

Nice to see you.

1. “Hello Sue!” “Hi Jan! Nice to see you.” 2. “…………………………” “Yes, can I help you?” 3. “Good morning.” “………………………… “ 4. “…………………………” “Fine, thanks.” 5. “Make yourself at home.” “…………………………” 6. “Good night.” “…………………………” 7. “What are you doing this weekend?” “…………………………” 8. “Thank you so much for helping.” “………………………” “……………………………” 9. “Have a nice day.” “…………………………….” 10. “Here‟s your beer.” “…………………………” 11. “See you next week.” “…………………………” Exercise 2. Complete each sentence. Use the words in the brackets. Use present simple or present continuous. 1. Richard (always, get up)……………………………………………………...….before 7.00. 2. Hurry up! The bus (wait)……………………………………………………….……..for us! 3. Where (we, go)………………………………………………….…? This is the wrong road. 4. My friends (not believe) ………………………………………………………….. my story. 5. Please be quiet! I (read) ………………….…………………….……a very interesting book. 6. (like, Susan)…………………………………..…………………………..…….horror films? 7. Maria (usually, sit) ………………………………………..……..… at the front of the class. 8. Carlos can‟t talk to you at the moment. He (have)……….……………….……….a shower. 9. Please wait for a moment, Jane. I (talk)……………………..……………..……… to Susan. 10. This bike (cost)………………………………………..………………….... a lot of money. Exercise 3. Complete each sentence using the correct present simple or present continuous of the verbs in the box. You may have to use some negative forms. belong

do

have

help

hold

move

use

1. In Monopoly, you ………………..around the board, buying houses and hotels. 2. ………………… you ………………this programme or can I turn the TV off? 3. Regular exercise …………………………. you to stay healthy. 4. I…………………………..my brother‟s guitar until I get a new one. 5. ……………………Simon always …………………the washing-up after lunch? 6. …………………….you ……………..any sweater in a larger size? 7. You……………………………….the kite right. Let me show you. 8. Dad……………………………to the local astronomy club. 1

watch

Exercise 4. Underline ten verbs in the wrong tense and rewrite them correctly. „One game I am loving is backgammon. You are throwing the dice and then you move your pieces around the board. It is seeming quite easy, but in fact you needing to be quite careful. When your piece lands on one of the other person‟s piece, you are taking it off the board and you send it back to the beginning. You are winning by getting all your pieces to the end off the board. Some people are preferring chess, but I am not understanding that game. Right now, I wait to have a game with my brother. He does his homework. I usually win, so I think he doesn‟t want to play a game with me!‟ 1………………………................... 2. …………………………….…… 3………………………………….. 4………………………………….. 5…………………………………..

6………………………………………… 7………………………………………… 8………………………………………… 9………………………………………… 10………………………………………..

Exercise 5. Read the text. Are the statements true (T) or false (F)? LIVING IN THE USA Roberto came from Acapulco to New York ten years ago. At first he missed everything – the sunshine, the food, his girlfriend. But now he has a successful business with his three brothers and his sister. They run a soccer store in New Brunswick. Roberto‟s girlfriend is now his wife, and they have two children who go to American schools. When asked why he came to the US, Roberto says without hesitation, „Because I want to work hard and be successful.‟ He certainly works hard. He‟s at the store all day, and then works as a driver in the evening. „That‟s why I like America,‟ he says. „You can be what you want.‟ „When I first came here, I didn‟t speak the language, and it was winter. It was so cold! There was snow! Now nearly all my family are here, not only in New York, but also in California, and in Texas. We meet about once a month and have a huge Mexican meal that takes about five hours! We‟re all happy here.‟ 1. Roberto came to New York with his girlfriend. 2. Roberto‟s sister lives in New York. 3. Roberto‟s family runs a sports shop. 4. Roberto is married. 5. Roberto didn‟t want to come to America. 6. Roberto has two jobs. 7. Roberto couldn‟t speak English at first. 8. The weather was a problem for Roberto. 9. All Roberto‟s family now lives in New York. 10. Some of Roberto‟s family want to go back to Mexico.

____ ____ ____ ____ ____ ____ ____ ____ ____ ____

Exercise 6. Use the words in bracket ( ) to complete the dialogues. Use present simple or present continuous. 1. Ann: Is the swimming pool open today? Jane: (No, they / get / things ready for the next competition.) …..…………………………………………………………………………….………

2

Ann: (How often / they have / these competitions?) ………….……………………………………………………………………….……… Jane: About three times a year. 2. Sam: (Why / that machine / not work / at the moment?) ………………………………………………………………………………….……... Chris: (I don‟t know, but a mechanic / mend it) ………………………………………………………………………………….……... Sam: (What / the machine / do / in fact?) (it / make / boxes?) ………………………………………………………………………………….……... Chris: (Yes, it/ make / boxes of all sizes.) …………………………………………………………………..……………………… 3. Mary: (Look! The Fosters / work / in the garden.) ………………………………………………..………………………………….……... Joe: (They / not / usually / work / on Saturdays) ..……………………………..…………….…………………………………………… Mary: Let‟s have a word with them. Hello, Sara, (you / get / ready / for the winter?) ………………………………………………………………………………….……... Joe: Yes, we / tidy up / the leaves) ………………………………………………………………………………….……... Mary: (We / always / try / to do it before the winter / come.) ………………………………………………………………………………….……... Exercise 7. Put in a form of have and have got. There is usually more than one possible answer. 1. Shall we …………………… a swim? 2. Trevor and Laura …………………. an argument at the moment. 3. ………………you ………….. a bike? 4. It‟s a nice big flat, although …………………….. (it/not) a balcony. 5. Peter usually …………………… a rest in the afternoon. 6. ……………..you…………….. any photos here to show us? 7. We cook in the evening. At lunch time I usually……………… a sandwich. 8. She ………………….... maths at nine o‟clock on Mondays. Exercise 8. Complete each sentence so that it has a similar meaning to the previous one. Include a form of have or have got. Example:

There‟s a girl with green hair over there. The girl over there has got green hair.

1. There‟s a pond at the end of the garden. The garden…………………………………………..end of it. 2. My tooth is hurting. I……………………………………………….….. toothache. 3. There is a CD with the magazine. The magazine ………………………. with it. 4. A man with a suitcase is getting on the bus. The man who is ……..……. ………………………………………......………………….. 5. I think that‟s my umbrella in your hand. I think…………………………………………………..…umbrella. 6. The material is very strong. The material ………………………………………….great strength.

3

Exercise 9. Write each sentence in a different way using the words given. 1. Tom doesn‟t want to go out because he has got a cold. Tom has got ……………………………………………………………………….…………. 2. I can‟t sleep because it is very hot. It is very …………………………………………………………………………………... 3. Jack is unhappy, so he doesn‟t want to see anybody. Jack doesn‟t ……………………………………….………….………….…………………. 4. Julia doesn‟t like apples, so she doesn‟t eat them. Julia doesn‟t……….……………………………………….………………………....…… 5. I want to lose weight, so I‟m eating less these days. I‟m eating…………………………………………………………………………………..… 6. The streets are crowded because there is a football match. There is a …………………………………………………………………………………….. Exercise 10. Join these pairs of sentences in three different ways using although, but and however. 1. She went on working. She was ill. …………………………………………………………………………………………… ………………………………………………………………………………………….... ……………………………………………………………………………………………… 2. Mathew has a car. He doesn‟t often drive it. …………………………………………………………………………………….…....… ……………………………………………………………………………………….….... …………………………………………………………………………………….….…… 3. Henry is a millionaire. He hates spending money. ………………………………………………………………………….…....….……… .………………………..……………………………………………………….……..….... …………………………….……………………………………………………....….…… 4. Laura wants to fly. She feels afraid. ……………………………………………………………………………………….……. …………………………………………………………………………………….….….... ………………………………………………………………………………….…..……… 5. You don‟t like Jessica. You should try to be nice to her. …………………………………………………….…………………………………...…... …… ……………………………………………………………..…………………….….... …………………….………………………………………………………………….….……

4

Mini Test 1 Part 1: Picture Description Listen and choose the statement that best describe the picture

(A) (B) (C)

1

2

(A) (B) (C)

(D)

(D)

(A)

3

5

(B) (C)

(D)

(A) (B) (C)

4

5

(A)

(B) (C)

(D)

Part 2: Questions and Responses Listen and choose the correct answer 1.

(A)

(B)

(C)

2.

(A)

(B)

(C)

3.

(A)

(B)

(C)

4.

(A)

(B)

(C)

5.

(A)

(B)

(C)

6

(D)

Part 3: Short Conversations Listen and choose the best answer to each question. 1. What feature does the man mention? (A) The woman's height (B) The woman's age (C) The woman's face (D) The woman's hair 2. What is the woman doing? (A) She is fixing the photocopier. (B) She is using the fax machine. (C) She is making photocopies. (D) She· is taking photos. 3. Which sentence is true? (A) Mary doesn't know the woman. (B) The photocopies are ready. (C) The woman is a new employee. (D) The man wants to make photocopies.

4. Why does the woman have a headache? (A) The room is very large. (B) The room is dark. (C) The man doesn't want to work. (D) The woman works too hard. 5. What warning does the man give the woman? (A) She must use less electricity. (B) She should relax more often. (C) She should not use a computer every day. (D) She should turn on the lights. 6. Who had a similar problem? (A) The man (B) The man‟s sister-in-law (C) The man‟s sister (D) The woman‟s sister

7

Part 4: Short Talks Listen and choose the best answer to each question. 1. Which of the following takes the longest time to get to the speaker's workplace (A) The subway (B) The bus (C) The bike (D) The taxi 2. How long does it take the speaker to walk to the subway station? (A) Ten minutes (B) Fifteen minutes (C) Twenty minutes (D) Fifty minutes 3. How does the speaker get to work in the winter? (A) By bus (C) By taxi (B) By train ` (D) By bike

4. What is the woman's job? (A) Designing cakes (B) Making wedding dresses (C) Planning weddings (D) Arranging flowers 5. Why is today an amazing day for the woman? (A) It is the woman's birthday. (B) It is her sister's birthday. (C) It is the woman's wedding. (D) She is planning her sister's wedding. 6. Which of the following is NOT true? (A) The woman designed a cake. (B) The woman will wear a nice dress. (C) The guest will dance. (D) The woman will get married.

8

Part 5: Incomplete Sentences Choose the word or phrase that best completes the sentence. 1. “---------------- are you staying?” “At the Park Hotel” (A) When (B) Why (C) Where (D) Why 2. I enjoy history lessons, --------------- I like the teacher. (A) so (B) although (C) because (D) however 3. “Do you usually see football --------------- on TV?” (A) courses (B) parties (C) plays (D) matches 4. Your shirt is nice, but it doesn‟t ---------------- your trousers. (A) suit (B) match (C) fair (D) miss 5. Carlos spends too much money ----------------- designer clothes. (A) in (B) for (C) on (D) at 6. Peter enjoyed the film, ----------------- the beginning was boring. (A) but (B) although (C) however (D) because 7. I am ----------------- my supervisor with job interviews these days. (A) help (B) will help (C) helped (D) helping 8. “----------------- bag is this?” “It‟s mine” (A) What (B) Whose (C) Where (D) Who 9. Jane ----------------- a computer lesson at the moment. (A) has (B) had (C) is having (D) has got 10. What a ----------------- jacket you‟re wearing! (A) love (B) lovely (C) loved (D) loving 11. We grew up together, ----------------- we know everything about each other. (A) so (B) because (C) but (D) although

9

12. You look worried. -------------- about what happened last night? (A) Do you think (B) Are you thinking (C) You think (D) You are thinking 13. “-------------- are these shirts”? “They are 15 pounds each” (A) How long (B) How much (C) How often (D) How many 14. Kate usually -------------- to class by bicycle, but today she went by bus because of the rain. (A) go (B) goes (C) went (D) going 15. Madeline must be very tired; she is -------------- on the sofa. (A) jumping (B) lying (C) awake (D) cleaning 16. We -------------- at the club every two months. (A) meet (B) met (C) have met (D) meeting 17. I don‟t think it will rain. However, if it --------------, turn off the machine. (A) don‟t rain (B) do rain (C) does rain (D) doesn‟t rain 18. “-------------- newspaper do you read?” (A) When (B) Where (C) Which (D) Why 19. She isn‟t English. --------------, she speaks English well. (A) But (B) Although (C) However (D) So 20. Could you give me -------------- pen, please? (A) other (B) another (C) whose (D) whom 21. We cook in the evening. At lunch time I usually -------------- a sandwich. (A) have got (B) have (C) are having (D) had 22. What did he -------------- her to do this morning? (A) say (B) tell (C) speak (D) question 23. I have a hot -------------- every morning and every evening. (A) rest (B) shower (C) break (D) water 24. -------------- a good journey. I hope the roads are clear. (A) To have (B) To have got (C) Have (D) Have got 10

25. He‟s a good friend, -------------- we don‟t meet often. (A) but (B) although (C) so (D) because 26. Claire dresses -------------- in New York- and wears her hair up. (A) different (B) differently (C) difference (D) differ 27. -------------- does it take you to go to school every day? (A) How far (B) When (C) Where (D) How long 28. The children enjoy playing in the sand and -------------- in the ocean. (A) to swim (B) swimming (C) swim (D) swam 29. Let‟s -------------- the art gallery first, and then have some lunch. (A) trip (B) journey (C) visit (D) watch 30. Collecting stamps gives me a lot of -------------- and helps me to relax. (A) qualification (B) rhythm (C) instruction (D) pleasure

11

Part 6: Incomplete Texts Choose the best word or phrase for each blank Questions 1 through 3 refer to the following letter. From: The principal To: All teachers I have recently received several ----------- from parents about teachers being late for class. 1. (A) complaints (B) complains (C) complain (D) complaining As you know, school policy is for all teachers to be in their classroom five minutes before the ------------- of a lesson. This sets a good example for our students. 2. (A) end (B) start (C) finish (D) introduction We cannot expect our students to be punctual if we ------------- late ourselves. All teachers must observe school policy 3. (A) to be (B) will be

(C) are (D) is

12

Questions 4 through 7 refer to the following information.

When most people think of movies, the city that comes to ------------ is Hollywood. 4. (A) their head (B) thinking (C) mind (D) to them

Hollywood is famous for its movie studios, and people often visit in the hopes of meeting a famous actor or actress. But these days, Hollywood is not the only world city famous for movie production. Another city that is becoming ---------- around the world is Mumbai, India. 5. (A) popular (B) famous (C) exciting (D) interesting In -----------, in the entertainment industry, Mumbai is known as Bollywood! 6. (A) truth (B) the case of (C) fact (D) movies

India's Bollywood produces an average of ten movies per day, and the movies are distributed throughout the world. All Bollywood movies follow the same ------------; 7. (A) story (B) script (C) idea (D) pattern there is a lot of singing, dancing, action, and romance, but there is no nudity or extreme violence. And Bollywood movies usually run for three hours or more!

13

Questions 8 through 11 refer to the following notice.

Looking for new players! The Mayfield High School Football team is ----------- for three new players. We are 8. (A) looking (B) hunting (C) researching (D) seeking currently leading in the High School league and hope to maintain our position. If you think you are ------------ to help take our team to the top for the tenth year in a row, come along to 9. (A) enough good (B) good enough (C) enough (D) good the football try-outs next Friday. We will be holding the try-outs on the athletics field between 7 a.m. and 11 a.m. If this is too early for you, then you ------------- the kind of 10. (A) isn't (B) are not (C) have not (D) are person we are looking for. All regular practices start at 7 a.m., and the team practices 5 days a week. We would like to encourage all students who made it to the short list last time -----------again. 11. (A) try (B) to try (C) trying (D) tried

14

Questions 12 through 15 refer to the following information

You are never too young or too old to get involved in sports and exercise. In fact the sooner you start to ------------, the better. According to some recent research, people who started sports 12. (A) exercising (B) sport (C) sporty (D) exercise at a young age were more likely to keep it up as they got older. This applied whether they were involved in -------------- sports or not. In fact, people who did sports just for fun when they 13. (A) competition (B) competitive (C) competing (D) competitions were younger were about 7% more --------------- to keep exercising when they were older. 14. (A) like (B) likeable (C) likely (D) liking Researchers believe this might be because some young people get burnt out from competition. However, even people who gave up sports because they felt burnt out tended to return to exercising a few years later. So start exercising now, if you want ----------- strong and healthy later! 15. (A) being (B) be (C) to be (D) to being

15

Part 7: Reading Comprehension Choose the best answer Questions 1 through 3 refer to the following notice.

Notice to All Employees As we enter the cold and flu season, the management wants to remind all employees to wash their hands after using the restroom and before returning to work. This is especially important for cooks, waiters and waitresses. As most of you are aware, germs, viruses, and bacteria are passed on mainly through hand contact. Here at The Happy Sandwich restaurant, hygiene and cleanliness are our number one priority. This policy will be strictly enforced. Thank you for your attention in this matter. The Management

1. What is the name of the restaurant? (A) The Management (B) The Happy Sandwich (C) The Winter Season (D) The Strict Policy

3. What is the restaurant‟s main priority? (A) Serving delicious food (B) Having the lowest price in town (C) Having the most polite staff (D) Hygiene and cleanliness

2. Which season is approaching? (A) Spring (B) Fall (C) Winter (D) Summer

16

Questions 4 through 7 refer to the following email .

Dear Mr. Thompson. This is just a short email to update you on your medical test results from your physical examination on January 10 th. Our records show that you took a blood pressure test. Your test this time showed that your blood pressure is doing quite well for a man your age. I am very pleased to note that your blood pressure is much lower that it was last time. It seems that the medication you have been taking is working well. Unfortunately, we do not have a record of the name of your medication on file. Can you remember the name of the medication that you are taking? If you could send an email to my nurse with the name of the medicine, that would be very helpful to us. His email address is [email protected]. Also, there is a note in your file stating that you want your medical test results sent to your insurance company. Which department do you want them sent to? Please let us know as soon as possible. Regards, Dr. Huxley

4. What kind of test did Mr. Thompson have done? (A) A stress test (B) A blood test (C) A high blood pressure test (D) A department test

6. What is true about Mr. Thompson‟s blood pressure? (A) It is extremely high. (B) It is very low. (C) It is non-existent. (D) It is normal.

5. Where will Mr. Thompson‟s medical test results be sent to? (A) His employer address (B) His insurance company (C) His doctor‟s office (D) The doctor‟s nurse

7. What information does the doctor need? (A) Mr. Thompson‟s email (B) The name of his medication (C) The nurse‟s email address (D) Mr. Thompson‟s age

17

Questions 8 through 11 refer to the following information.

Sugar Dairy, Meat Fish, Eggs Vegetables and Fruit Bread, Rice, and Cereals

This is a chart of the Food Pyramid. The Food Pyramid serves as a basic guide to making healthy food choices. If you take a look at the guide you can easily see which food groups you should be eating. According to the pyramid, most of your daily food should consist of bread, rice, and cereals. These are the foods in the bottom lever of the pyramid. Almost equal to these, but not quite as much, should be vegetables and fruit. Therefore, you can feel free to go ahead and eat lots of fruit and vegetables every day. Serving of meat, fish, eggs, and dairy foods, which include milk and cheese, should be much smaller. Sugar should be the smallest portion of all. Of course, your daily nutritional needs will vary according to your activity lever and life style. You do not have to follow the Food Pyramid, but it is a good way to be sure you will get the healthiest benefits from your daily food. If you keep a copy of the pyramid stuck to the door of your refrigerator, it will remind you to plan your daily meals wisely every time you go into the kitchen.

8. Which of the following are daily products? (A) Milk and cheese (B) Fish and eggs (C) Sugar and spice (D) Bread and rice

10. Which food group has the second smallest serving suggestion? (A) Sugar (B) Dairy, Meat, Fish, and Eggs (C) Vegetables and Fruits (D) Bread, Rice, and Cereals

9. What is the purpose of the Food Pyramid? (A) To sell food (B) To help make healthy food choices (C) To show which foods are bad for you (D) To help remember the names of foods

11. What affects your daily food needs? (A) Your likes and dislikes (B) The Food Pyramid (C) Your lifestyle and activity (D) Small portions

18

Questions 12 through 15 refer to the following article.

12. Who are these instructions for? (A) (B) (C) (D)

14. How many methods of listening to music are mentioned in the text? People who want to listen to music. (A) None Musicians (B) 3 Credit card companies (C) 60 MP3 players (D) 4

13. How can people pay for the music? (A) They can send a check. (B) They can use a credit card. (C) They can borrow money. (D) It does not say.

15. How can people find a song? (A) Go to large music store (B) Use the search engine on download sites (C) Listen to the radio (D) Ask Melody Magazine

19

Revision 2: Units 3 and 4 Practice 2 Exercise 1. Put the word in the right form. 1. The thief tried to ……………………… the door but he couldn‟t (lock) 2. He died in a ……………………………circumstance and the policemen haven‟t finished the investigation on his death. (mystery) 3. She is scared when thinking about the …………………house she lived in 5 years ago.(haunt) 4. We apologize for the late ……………… of the train.(arrive) 5. They deserved a gold medal for their …………………..performance. (amaze) 6. He was soon…………………… of all dangers.(forget) 7. Do your parents take ………………….. in your friends? (interesting) 8. The goods are on sale today, so we can buy a lot of ……..…………….items.(expensive) 9. She‟s a …………………person. She always disturbs other people by her gossips. (talk) 10. The police are trying to have a ................................of the suspect.(describe) Exercise 2. Put the verbs in the past simple or past continuous. 1. I .....................(go) to the cinema yesterday and ...................(see) the new James Bond film – Skyfall. It ......................... (be) great. 2. At 4:30 last Friday afternoon, I ............................ (have) an English lesson. 3. She .................. (write) a letter to her daughter last night when there ................ (be) a phone call. She ................... (pick up) the receiver and .................(hear) her daughter on the line phoning all the way from Africa! She ............... (not need) to finish the letter because she ................ (tell) her all the news over the phone! 4. They .................... (ask) us last week to go on holiday with them. We ............. (say) “yes”! 5. As Tom ................ (walk) home from work last week, he ............. (see) a bag at the side of the path. It .................. (seem) to be moving! He .................. (go) closer and ................... (look) inside. There .................... (be) two small kittens and a note saying: “Please take us home and feed us – we need a loving home!”. 6. This time last year I .......................(lie) in hospital after having a serious accident. 7. We .................... (travel) into town on the bus. We ...............(have) to change twice but it ......................(not take) us very long. We ..................(be) there in 20 minutes. However, coming back home .................. (take) much longer!!! 8. Last winter it .............................. (snow) a lot. It .................... (be) also extremely cold! 9. I .................... (get up) really late this morning and .................... (have) to run for the bus. The bus driver .....................(see) me running and ...................... (wait) for me . It ............ (be) very nice of him. 10. While I ................................(write) this exercise for you, my three cats ......................(sit) around the computer. They .........................(wait) patiently for me to finish!

20

Exercise 3. Use the words and phrases in each item to build a complete sentence. You can make changes or additions if necessary. 1. They / steal / painting / local museum / and/ hide / away / police. ………………………………………………………………………………………………… 2. When / I / watch / paintings / Dali / he / come / talk / me. …………………………………………………………………………………………............... 3. How many / plane / there / air show? ……………………………………………………………………………………………… 4. How much / milk / you / need? ………………………………………………………………………………………………… 5. Jack / spend / two / hour / do / homework / yesterday. ....................................................................................................................................................... Exercise 4. Choose the best answer. 1. Would you like ..................... coffee? (few, some, little) 2. I haven‟t got ....................... money. (some, any) 3. ...................... idiot keeps calling me on my cell phone. (some, any) 4. There was ........................ hope for the patient. (little, few) 5. There are ...................... takers for the new car. (few, little) 6. ......................... members were absent from the meeting. ( a few, a little) 7. Foreign languages are difficult, and ………………people learn them perfectly (few, a few) 8. I have ………….… time to read newspapers, but I have no time at all to read books. (little, a little) 9. Is there...................wine in the bottle? (some, any) 10. Can I have ....................beer, please? (some, any) Exercise 5. Put a, an or no article into the correct gap. 1. Is this ...............book you were telling me about? Yes, it is about ................life of Queen Elizabeth 2. Is there .................supermarket here? There are several, ..................nearest one is just down ..........................road on ..................... right. 3. We always stay at .................. Kensington Hotel because it is ...................only one with facilities for .................disabled. 4. ....................Swiss Alps are ......................good place to go if you like ................skiing. There is usually plenty of ....................snow during ....................winter months. 5. Just look at ....................snow! It seems to be just ..................right kind of ....................... snow for ....................skiing. 6. ...................English people are forever complaining about ...................weather. 7. This is ....................toughest steak I have ever eaten. It is ..................last time I eat in this restaurant. 8 . ....................otter is found only in .................north of Britain. 9. He writes ...................... books about ....................natural history in ................... general. 10. …………….sun rotates around …………….earth.

21

Exercise 6. Use the words in the box to complete the conversation. a lot of

any (x2)

anything

everyone

many

some

something

nothing

How (1) …many…people are coming to the party? Well, there are (2) ……..…… my friends – about thirty. Have we got (3) …………………to drink? There‟s (4) …………………. wine and beer in the fridge. We need orange juice, too. I‟ll go and buy some. Have you got (5) …………… money? Have a look in my purse. There‟s (6) ………………………in your purse! I‟ll use my credit card. Do we need (7)……………….. bags of crisps? B No, we don‟t. There are twelve bags in the cupboard. A OK, I‟ll go to the shop now. B. I‟ve just remembered (8) ……………. – (9) …………….. is arriving at 6.00 so you need to hurry. A B A B A B A

Exercise 7. Fill one suitable word into each gap of the text below. Sarah Fleming (1) .................................a horrible experience while she was on holiday last summer in New York City. (2) .................................. the airport, as she was waiting to catch the airplane to return home, her dress (3) .................................. dirty and she didn‟t have any spare clothes in her bag to change. She tried to buy (4) ................................. to wear but all her credit cards were missing. She knew that she was (5) ................................. trouble because she didn‟t have (6) ................................. money on her. She decided to sit down and got her head clear in order to see what she (7). .................................. would do. As she was sitting in the waiting room, she saw (8). .................................... old friend from college. She got up and walked up to her to say “Hi”. While they were (9). ................................. about the old times, Sarah heard her name over the loud speaker. An airport security officer had found her purse. Her problem was resolved. Shortly after her flight was (10). ................................. to depart. Exercise 8. Rewrite each of the following sentences so that its meaning stays the same. 1. How long is it since they bought the house? When ......................................................................................................................................... 2. When you phoned me, it was my lunch time. When you phoned me I ............................................................................................................ 3. During dinner, the phone rang. While I ...................................................................................................................................... 4. We lived in London till 1995. We used .................................................................................................................................... 5. I really want to know what you were doing when he came. What .......................................................................................................................................?

22

Exercise 9. There is one mistake in each sentence. Find the mistake by circling the correct answer A, B, C or D.

1. The poor child thanked us for giving him food and moneys. A B C D 2. The doctor carefully is examining the patient who had an accident last night. A B C D 3. New product were on display in the electronic exhibition in Jakarta Expo Centre a week ago A B C D 4. Because of the badly weather, we had to cancel our camping trip. A B C D 5. Watch out! Anyone is following you on the wrong side of the road! A B C D

Exercise 10. Complete the sentences with during, while, or for. 1. I went sailing … during… my holiday. 2. We met …………..…..we were at university. 3. We lived in Zanzibar ………………….. two years. 4. We‟re going away ………………….two months this year. 5. It is very hot ……………….... the day. 6. …………………. we were waiting, we saw some old friends. 7. Jane had a headache …………………. the examination. Exercise 11. Write at, on, in, or ✗ (nothing). 1. I get up …at…. 6.30 a.m. 2. The meeting‟s …………..Monday. 3. Why didn‟t you come to visit me ……..….. last year? 4. We‟re seeing them ……………next week. 5. Let‟s go on holiday ………….August. 6. My birthday‟s ………….5th March. 7. Let‟s meet .....................Wednesday afternoon ...................... half past two. 8. I‟m going away ......................Easter, so how about the next week? 9. What are you going to do ...................the weekend? 10. If we ever go camping again, it‟s going to be .................. summer, not autumn.

23

Mini Test 2 Part 1: Picture Description Listen and choose the statement that best describe the picture

1

(A) (B) (C) (D)

2 (A) (B) (C) (D)

3

(A) (B) (C) (D)

24

4 (A) (B) (C) (D)

5 (A) (B) (C) (D)

Part 2: Questions and Responses Listen and choose the correct answer 1.

(A)

(B)

(C)

2.

(A)

(B)

(C)

3.

(A)

(B)

(C)

4.

(A)

(B)

(C)

5.

(A)

(B)

(C)

25

Part 3: Short Conversations Listen and choose the best answer to each question. 1. How often does Bill lift weights? (A) Twice a week (B) Three times a week (C) Four times a week (D) Every day 2. Which of the following does Bill NOT do? (A) Go swimming (B) Play basketball (C) Go jogging (D) Lift weights 3. What is true about the woman? (A) She thinks Bill should exercise more. (B) She thinks Bill looks healthy. (C) She goes swimming every week. (D) She runs two times a week.

___________________________________________________________________________

4. Why does the woman want to do yoga? (A) Her friend does yoga, and she wants to join, too. (B) She wants to become an instructor. (C) She wants to improve her health. (D) She knows that it is cheap. 5. What are two good things about The New Health Yoga Centre? (A) It is not far away and the instructors are friendly. (B) It is close and very cheap. (C) It is near, and the instructors are good. (D) It is clean and cheap. 6. Why should the women join this month? (A) There is a discount. (B) Her friend is joining. (C) A new class is starting. (D) She knows the instructor.

26

Part 4: Short Talks Listen and choose the best answer to each question. 1. What is John‟s job? (A) He is a soccer player. (B) He is a soccer coach. (C) He is soccer referee. (D) He is a sports writer. 2. What is true about John? (A) He has refereed before. (B) His first game is next week. (C) His first game is next month. (D) He hasn‟t played soccer for long. 3. How long has John been playing soccer? (A) 2 years (B) 3 years (C) 4 years (D) 5 years ___________________________________________________________________________

4. How old are the children who play T- ball? (A) Under 10 years old (B) Over 10 years old (C) From 11 to 15 years old (D) All ages 5. What is special about T-ball? (A) The ball is thrown to the hitter. (B) The ball is placed on a stand, and the child to hit it. (C) They learn many complicated rulers. (D) Children exercise very hard. 6. What is true about baseball? (A) It is more popular with girls. (B) Some parents coach during practice sessions (C) Children younger than 15 do not play it (D) They do not play it in middle school

27

Part 5: Incomplete Sentences Choose the word or phrase that best completes the sentence. 1. Thank you for your ----------------- invitation. Unfortunately, I can‟t come. (A) kind (B) kindly (C) unkind (D) kindness 2. We had a ---------------- meal and then went to the theatre. (A) lovely (B) love (C) loving (D) loneness 3. The exam was ---------------- difficult. I couldn‟t do any of it. (A) real (B) unreal (C) really (D) reality 4. There was an accident, but no one was---------------- injured. (A) serious (B) seriously (C) unserious (D) unseriously 5. I have ----------------- £3.52 in my pocket. Not much, is it? (A) exactly (B) exaction (C) exact (D) exactness 6. ----------------we got home, we listened to some music. (A) Ago (B) Last (C) When (D) If 7. The judge didn‟t believe him and sentenced him ----------------nine years. (A) in (B) to (C) into (D) at 8. Larry immediately said, "I can't go, nor ----------------- want to go." (A) I (B) do (C) I do (D) do I 9. I don‟t have ------------------ on the hotels of the island. (A) much information (B) many informative (C) much informs (D) many information 10. I didn‟t go home ---------------- weekend because some friends came to stay. (A) ago (B) last (C) when (D) if 11. Have you got ---------------- brothers or sisters? (A) a (B) many (C) some (D) any 12. I went shopping but I didn‟t buy ---------------(A) someone (B) something (C) anyone (D) anything 28

13. Don‟t let --------------- touch my computer while I‟m out. (A) someone (B) something (C) anyone (D) anything 14. How ---------------- money do you have? (A) cost (C) much

(B) long (D) many

15. I‟ll be ready in --------------- minutes. (A) a little (C) a lot of

(B) a few (D) much

16. I have ----------------close friends. Two or three. (A) a (B) a lot of (C) a few (D) a little 17. Listen! I‟ve --------------- important to tell you. (A) somebody (B) something (C) anybody (D) anything 18. What ---------------- horrible day! (A) a (C) the

(B) an (D) Ø

19. When we got to the meeting, there was ---------------- to sit because it was so full. (A) somewhere (B) anywhere (C) everywhere (D) nowhere 20. There‟s ----------------- I want to tell you. Can you keep a secret? (A) nothing (B) everything (C) something (D) anything 21. She is very practical. She wants to marry a ----------------(A) a million (B) millionaire (C) millionaires (D) millions 22. These policies are unlikely to prove -------------- with middle-class voters. (A) popular (B) popularity (C) popularly (D) popularize 23. It‟s -------------- in America to eat turkey on Thanksgiving Day. (A) tradition (B) traditionally (C) traditional (D) traditionalism 24. They were really -------------- about the result of the match. Their favorite team became champions for the first time. (A) excitement (B) exciting (C) excited (D) excite 25. You should be --------------- about which information to include in the report. (A) selective (B) selection (C) select (D) selected 29

26. There's a world of -------------- between liking someone and loving them. (A) different (B) differ (C) difference (D) indifference 27. We are looking for people who would be willing to -------------- in the group's work. (A) assisting (B) assistant (C) assistance (D) assist 28. He is a very ---------------- person. (A) society (C) sociably

(B) sociable (D) socially

29. It's ---------------- cold. (A) terror (C) terrible

(B) terrify (D) terribly

30. They have certainly studied ---------------- to pass the test. (A) hard enough (B) hardly enough (C) enough hard (D) enough hardly

30

Part 6: Incomplete Texts Choose the best word or phrase for each blank Questions 1 through 4 refer to the following announcement.

Attention, all airline -------------- Please do not leave your bags unattended while you use the 1. (A) people (B) passengers (C) airplanes (D) users restroom or wait in line for your boarding pass. There have been several cases of theft in the airport lounge, so we -------------- keeping your bags with you at all times. 2. (A) tell to you (B) say to you (C) recommend (D) order If you are missing any of your luggage, please report it to security personnel as soon as possible. You can also come to the lost and found area to -------------- your bags there. 3. (A) search for (B) look at (C) keep (D) throw away It is advisable to keep a--------------on your bag, as that will help us to more quickly assist 4. (A) airline ticket (B) driver license (C) name tag (D) hand you if the need arises. Thank you for your cooperation.

31

Questions 5 through 8 refer to the following advertisement

Summer --------------! Don‟t miss out on this special limited- time only travel package 5. (A) travelers (B) tour takers (C) vacation (D) time to beautiful Hawaii. This travel package includes round trip, air fare and hotel reservations for 3 nights and 4 days at the Kawakawa Hotel located right -------------- the white, sandy beach of Hilo. 6. (A) in (B) at (C) on (D) on top This special package also includes free buffet, breakfast and the use of a rental car with unlimited mileage, so you can see the beautiful main island of Maui at your own ----------7. (A) speed (B) distance (C) leisure (D) time This -------------- package will be available for one week only, so see your travel 8. (A) common (B) Hawaii (C) big (D) one -of- a-kind representative at Sunland Travel soon. You won‟t want to miss it!

32

Questions 9 through 12 refer to the following letter

Dear Grandma, I just wanted to -------------- you this email updating you on my trip, so far. 9. (A) send (B) give (C) tell (D) prepare for As you know, I ------------from Alaska on my motorcycle with the intention of riding down 10. (A) escaped (B) went out (C) set out (D) will go from Alaska through Canada, through the West Coast of the United State and down to the bottom tip of Mexico. Right now, I am in Vancouver, Canada. Vancouver is a really beautiful city. It is very clean and the air is very --------------. 11. (A) smoggy (B) dirty (C) fresh (D) airy I will probably stay here for two more days, and then continue down to Seattle, Washington. I have a friend in Seattle that I will ------------- with for a week, and after that, I will continue 12. (A) stay (B) meet (C) be (D) see on my trip. I will write again soon. Love, Freddy

33

Questions 13 through 15 refer to the following information

CHOOSING A COLLEGE -----------a college is one of the most important decisions you will have to make. The 13. (A) To choice (B) Chosen (C) Choosing (D) Having chosen following suggestions should help you make an informed decision. Visit the school you are considering. While you are at the school, take some to look at the equipment and facilities. Talk to lecturers and ------------- students. Ask questions that will give you first- hand 14. (A) now (B) current (C) presently (D) immediate knowledge about the school: do the instructors seem knowledgeable? What is the students‟ opinion of the instructors? What -------------they like most and least about the school or 15. (A) do (B) are (C) had (D) can program? Finally, look at several schools that offer similar programs. Compare program length, schedule, cost, transferability of course credit, financial aid availability and any other factors that are important to you.

34

Part 7: Reading comprehension Choose the word or phrase that best completes the sentence Questions 1 through 4 refer to the following advertisement .

Obituary Audrey Sarah lves Beloved wife of Albert, mother of Joan and George It is with great sadness that the Ives family notes the passing of Audrey Sarah Ives. Mrs. Ives, aged 82, died at her home on Tuesday morning, having been in bad health for the past year. She was a well-known amateur artist and often exhibited her best works at the Greenwich Art Society. Her paintings toured throughout Europe, and she even painted a portrait of the Queen. Her greatest achievement was the Alford Prize, which she won in 1985. She was the younger sister of the well- known sculptor Annie Taverner. Although not as well known as her sister, many think her talent was greater. Mrs. Ives was also a concert violinist in her younger days and performed with the Pinedale Orchestra. She also served in the Peace corps for five years as a nurse's aide in Rwanda where she was awarded the Queen's Medal for Valor when she assisted an injured tourist who was attacked by a tiger. Mrs. Ives fought the tiger with her bare hands and then provided lifesaving medical attention to the tourist. In addition to her husband Albert, she is survived by a son George, a daughter Joan and four grandchildren. Flowers can be sent to the Pine Grove Funeral Home in Spring Valley.

1. Where would you expect to see this information? (A) In a romance novel (B) In a newspaper (C) In a TV magazine (D) On a poster 2. How long was Mrs. Ives ill? (A) 82 years (B) All her life (C) 18 months (D) One year 3. What did many people say about Mrs. lves, with regard to her sister? (A) Mrs. lves didn‟t like painting. (B) Mrs. lves was a more famous painter. (C) Mrs. lves was a better artist. (D) Mrs. lves was richer. 4.

What musical instrument did Mrs. lves play? (A) Cello (B) Violin (C) Guitar (D) Piano

35

Questions 5 through 8 refer to the following advertisement Stay at the Svinkaire Hotel in Oslo- Winter Wonderland Special A little taste of luxury at affordable prices. The Svinkaire Hotel and Restaurant is the oldest family-run homeland restaurant in Iceland. For over one-hundred years the Svinkaire family has been providing the best service and comfort that the Northern Hemisphere has to offer. Escape the cold with a Svinkaire Winter Wonderland Special Deal. From December 15'" through March 15'", we are offering a discount rate on all our rooms. Up to 50% cheaper than our regular rates! Enjoy our top of the range sauna cabins, made of the finest quality wood. Or why not release your stress in one of our aromatherapy steam rooms? If you like sports why not try your hand at reindeer wrestling? Or if you are an artist at heart, you could try ice sculpture taught by Iceland‟s leading ice sculpture instructor, Sven Boorntorg. Our rooms are all equipped with TVs and DVD players. The hotel offers a full range of DVDs to rent, including the latest releases. We also provide Internet access. The Winter Wonderland Specialincludes your choice of a free buffet breakfast or dinner, and an evening sleigh ride to see the snow-capped mountains of Blintstroom. Call 92 3-8 72 www.svmka1re.com.

5. How long will this deal last? (A) One month (B) Three months (C) One week (D) One year 6. What is the hotel offering for free? (A) DVDs (B) A room (C) One meal (D) A reservation 7. Which of the following is NOT mentioned in the advertisement? (A) Internet access. (B) Saunas. (C) Room service. (D) Breakfast 8.

How long has the Svinkaire Hotel? (A) For 10 years (B) For 100 years (C) Over 100 years (D) Over 100 days

36

Questions 9 through 12 refer to the following article. How often do you use a vending machine? If you are like m ost people, you probably use one every day. They can be found all over the world. In America, these machines make billions of dollars every year. Vending machines sell almost e very kind of product that you can think of. Some of them s e l l drinks, while others sell candy and snacks . For workers who don't have any time for lunch, many business es and public buildings provide vending machines that also sell soups, salads, sandwiches, fruits, and desserts. Vending machines may a l s o offer p r o d u c t s such as stamps, train and bus t i c k e t s , newspapers, and other kinds of merchandise. There are even some vending machines that sell flowers

Vending machines have been a great success since they were first introduced to the marketplace. What are some of the reasons for this success? First, they save time. You may not have time to go to the store during your break to buy a soda, but you can go to the machine in the corridor. Secondly, they are open 24 hours a day. You can buy something even when all the local stores are closed. Another reason is that customers usually don‟t need to wait in line. At the store there may be many people in front of you and you have to wait for the cashier to serve you, but at the vending machine your transaction is all finished in moments. Lastly, the machines save companies money because there is no need for salesclerks and cashiers. The machine will do all the work and it doesn‟t need to be paid a salary or a wage.

9. What do these machines provide? (B) Cashiers (A) Customers (C) Salesclerks (D) Products 10. What makes these machines successful? (A) They are used only in businesses. (B) People can use them any time. (C) The cashiers are very friendly. (D) You can eat there. 11. How do these machines save companies money? (A) They are found in business buildings. (B) There is no need for cashiers. (C) They don‟t make any money. (D) Workers don't have time to eat. 12. Which of the following is NOT mentioned as something you can buy from a vending machine? (A) Flowers (B) Tickets (C) Eggs (D) Candy

37

Questions 13 through 15 refer to the following advertisement. Price Savers Supermarket For one week only! th Monday September 5th to Saturday September 10 We are pleased to announce another of our super bargain events. Price Savers always saves you money, and now you can save even more. Special offers; Dairy products Monday - buy one liter of milk, get one free Tuesday - Cheddar cheese 50% of recommended retailer's price Wednesday - 6 fruit yoghurts for the price of 4. Thursday - milkshake (individual 200ml cartons) 20% off until 3 p.m. Friday - Price Savers' own brand real dairy ice cream (vanilla, chocolate or toffee crunch) 30% of regular price Saturday - your choice of any one of the above! Sunday - store closed And that's not all. There's more! Baked goods Monday - Muffin variety 6 packs, buy 2 get second pack 50% off Tuesday - Baguettes, 3 for the price of 2 Wednesday - Baguettes, 3 for the price of 2 Thursday - buy one ready- sliced white loaf, get another for free Friday - 1 pound cake free to the first 100 customers Saturday -your choice of any of the above (offer excludes free pound cake) Sunday - store closed- saving you this much money is hard work! All offers are limited to one per customer per day. Sorry, but you can't enjoy the same offer twice in t h e s a m e day. Price Savers Supermarket -where prices and quality count. 13. How many days will this special offer free cake? (A) (B) (C) (D)

2 7 50% off 100

14. What is the weekend special offer? (A) There is no special offer. (B) Customers can choose the offer they prefer. (C) Customers can choose free fruit and vegetables. (D) It does not say. 15. How many customers can receive a free cake? (A) All of them (B) 100 (C) 8 for the price of 6 (D) One

38

Revision 3:

Units 5 and 6

Practice 3 Exercise 1. Put in the correct form of the verb in brackets. 1. We‟ve decided……………….…. (go) to the beach. 2. I stopped ……………………. (play) tennis when I got married. 3. I‟d love …………………… (visit) China. 4. What do your children want………………….. (do) when they leave school? 5. Ann is good at writing, so she is hoping ……………….. (work) for a newspaper. 6. Have the men finished………………………. (repair) the roof yet? 7. Do you remember ………………… (go) to Germany when you were three years old? 8. I look forward to …………………… (see) when I come next week! 9. I really enjoyed …………………. (listen) to those MP3 you sent me. Thanks. 10. I‟m thinking of …………………… (travel) around the country with Peter next summer. 11. I‟d like…………………. (lie) on the beach today. It‟s too hot to do anything else. Exercise 2. Underline the correct adjective in each sentence. 1. 2. 3. 4. 5. 6. 7. 8.

My work is tiring / tired. It made me tiring / tired. I didn‟t think you were interested / interesting in the ancient history. The first part of the film is really excited / exciting. If you felt bored / boring, why don‟t we go to the cinema? You look worried / worrying. Is anything the matter? Her action made me embarrassed / embarrassing. I don‟t know why my daughter loved that frightened / frightening dog. That was shocked / shocking news.

Exercise 3. Put each verb in brackets into a form of going to, will or present continuous. More than one answer may be possible. 1. Have you heard the news? Harry (join) is joining / is going to join the Army. 2. Sorry to keep you waiting. I (not be) …………….. long. 3. According to the weather forecast, it (snow) …………….. tomorrow. 4. I‟m sorry I can‟t meet you tonight. I (go out) …………. with my parents. 5. Careful! You (knock) …………. that jug off the table. 6. In 50 years‟ time, most people (probably ride)…………..bicycles to work. 7. Our teacher (give) ………….. us a test tomorrow. 8. I (go) ………….. to Manchester at the end of the next week. 9. Look out! You (hit) …………….. that tree. 10. I think our team (probably win) ……………

39

Exercise 4. Complete the second sentence so that it has a similar meaning to the first sentence. 1. My party is on Thursday. I’m having a party on Thursday. 2. Tomorrow‟s weather forecast is for rain. It‟s ……………………………………….tomorrow. 3. I predict a victory for our team. I think …………………………………………. win. 4. Tomorrow I‟ll be absent, Mrs. Jones. I ……………………………………… here tomorrow, Mrs. Jones. 5. Terry intends to finish painting the kitchen this evening. Terry …………………………….. painting the kitchen this evening. 6. Meet me outside the station at 5.30. I…………………….. outside the station at 5.30. 7. What‟s our arrangement for lunch? Where ……………………………………..for lunch? 8. Everyone expects lots of tourism in this country next summer. Every one thinks a large number of tourist ……………………. this country next summer. Exercise 5. Write the questions. Use What ... like? / like. A: B: A: B: A: B: A: B: A: B: A: B:

(1)…Did… you …like… Amsterdam? Yes, we liked it very much. (2) …………….. Amsterdam …………………. ? It‟s smaller than London so it‟s easy to find places. (3) ……………………. the food? Yes, I did, but I like Spanish and Italian food more. (4) ……………. it …………………..? It‟s a bit like British and German food – meat, potatoes, and vegetables. And lots of cheese! (5) ……………the countryside …………………? It‟s flat! It‟s the flattest place I‟ve ever been to. Where did you stay? (6) ………………your hotel ………………… ? It was quite small but nice and very central.

Exercise 6. Write polite replies. 1. His wife is so boring! Yes, she’s not very interesting. 2. That cat‟s stupid. Yes, …………………………………………………………………… . 3. Their house is dirty. Yes, ………………………………………………………………… 4. They are very mean. Yes, ………………………………………………………………….. 5. It was a horrible present. Yes, ……………………………………………………..……. . 6. She‟s always rude. Yes, ………………………………………………………………… Exercise 7. Complete using the correct form of the words in brackets. I think this is the (1)……………….. (bad) job I‟ve ever had. My last job was much (2)…………….. (good) than this one. I had a lot (3) ……………….. (little) work there and my boss was really nice. My boss here is the (4) ……………. (strict) in the whole firm, and the working day is (5) ………………… (long) than in my last job, too. The (6)…………….. 40

(good) thing is that the office is (7) ……………. (close) to my house than the old one. At least now I get home (8) …………….. (early) than I used to. Exercise 8. Complete the passage using the correct form of the words in the box.

Exercise 9. Complete the second sentence so that it has a similar meaning to the first sentence. 1. David is a better runner than Paul. Paul is not……………………………………………………………………………….. 2. Nobody in the class is taller than Anna. Anna is the ……………………………………… …………………………………….. 3. I haven‟t written as much as you. You‟ve written………………………………………………………………………….. 4. Jane‟s hair isn‟t as long as Sophie‟s. Sophie‟s hair is………………………………………………………………………….. 5. No student in the school is noisier than I am! I‟m the……………………………………………………………………………………. 6. This exhibition is much more interesting than the last one. The last exhibition was not……………………………………………………………….. 7. Kate ate much less than Helen did. Kate didn‟t ……………………………………………………………………………….. Exercise 10. Complete using the word in the box. You have to use some words more than once. Where

which

who

whose

1. There‟s a film tonight………………….I really want to see. 2. Do you know any restaurants…………..……..they serve vegetarian dishes? 3. What‟s the name of that writer ………………………. book won a prize last week? 4. The film in …………………the star is Tom Hanks, is based on a book by Dan Brown. 5. I‟ve never met anyone …..……………...….. mother was famous. 6. The newspaper is going to publish the article…………………..my sister wrote. 7. Lorenzo,…………..…..… is from Spain originally, has lived here for about ten years. 8. Carla, ………..……... parents are from Mexico, was born in the UK. 9. Here‟s a photo of the hotel……………......we stayed. 41

Exercise 11. Rewrite as one sentence using a relative clause. 1. My grandfather was an airline pilot. He is sixty-five years old now My grandfather, who is sixty-five years old now, was an airline pilot. 2. “Friendly People” is a comedy. It‟s my favourite programme. ………………………………………………………………………………………… 3. My friend Michael often comes to play with me. He hasn‟t got any brothers and sisters ………………………………………………………………………………………… 4. My sister loves wearing hats. Her hair is brown. ………………………………………………………………………………………… 5. New York is an enormous city. It‟s where I was born ………………………………………………………………………………………… 6. This CD is scratched. I only bought it yesterday ………………………………………………………………………………………… 7. My brother George has got some great shirts! He hates me borrowing his clothes. ………………………………………………………………………………………… 8. Our neighbours have never invited us to dinner. Their house is directly opposite ours. ………………………………………………………………………………………… Exercise 12. Complete each sentence with one of the words from the box. Use each word one only. extremely

fast

interesting

quite

shocked

hard

terrible

well

hardly tired

1. Jack dances very……………….and never steps on people‟s feet. 2. Alan was so tired that he could……………..keep his eyes open. 3. That is so…………………news. Thank you for informing me! 4. George was driving too……………..and was stopped by the police. 5. It‟s not a wonderful film, but it‟s ………………..good. 6. My father was……………..after my mother left him alone. 7. Gina worked very………………..and was given an extra holiday. 8. When I realized I hadn‟t paid for the coat, I felt……………………. 9. I can‟t afford to buy that bike because it‟s……………………expensive. 10. People often feel………………after a hard working day. Exercise 13. Put one word in each space. 1. Our team is ……………. good……………… your team. They‟re both the same. 2. This is one of ………………… ………………….. famous paintings in the world. 3. You‟re not a safe driver! You should drive ………………… slowly. 4. Ann is taller……………….Mike but their son Dave is………………. tallest in the family. 5. What an awful book. It‟s one of ………….….. ….…………. interesting I‟ve ever read. 6. It makes no difference, because this road is …………..... ……... ……. bad ……..…… that one. 7. Today is ……………... cold …………….. yesterday, so I‟m wearing my shorts.

42

Mini Test 3 Part 1: Picture Description Listen and choose the statement that best describes the picture.

1

2 (A) (B)

(C)

(A) (B)

(C)

(D)

(A) (B)

(C)

(D)

(D)

3

43

(A) (B)

4

5 (A) (B)

(C)

(D)

Part 2: Questions and Responses Listen and choose the correct answer 1.

(A)

(B)

(C)

2.

(A)

(B)

(C)

3.

(A)

(B)

(C)

4.

(A)

(B)

(C)

5.

(A)

(B)

(C)

44

(C)

(D)

Part 3: Short Conversations Listen and choose the correct answer. 1. Where does the woman want to go? (A) To the library (B) To the post office (C) To the bookstore (D) Next door 2. How long will it take to get there? (A) No more than ten minutes (B) Fifteen minutes (C) One hour (D) At least ten minutes 3. When does the woman need to be in the city center? (A) In half an hour (B) In sixty minutes (A) In two hours (C) In forty minutes __________________________________________________________________________ 4. Where is the man going? (B) To work (C) To school (D) To the store (E) To a funeral 5. What does the woman want? (A) Chinese food (B) A book (A) Some eggs (C) Some milk 6. How many kinds of items will the man bring home? (A) One (B) Two (C) Three (D) Four

45

Part 4: Short Talks Listen and choose the best answer to each question. 1. Why does the man like his job? (A) It is very easy. (B) He works short hours. (C) He likes his co-workers. (D) It doesn‟t feel like a job to him. 2. What kind of place does the person want to sing in? (A) In a bar (B) In a restaurant (A) In the theater (C) In the park 3. Who does the man sing with? (A) His cousin (B) His father (C) His brother (D) His friend

4. Who is this ad aimed at? (A) People with new cell phones (B) People with old computers (A) People with old cell phones (C) People with new computers 5. When will this sale take place? (A) On Monday (B) On Wednesday (C) On Thursday (D) On the weekend 6. What should you be if you missed this sale? (A) Angry (B) Sad (A) Sleepy (C) Crazy

46

Part 5: Incomplete sentences Choose the word or phrase that best completes the sentence. 1. I don‟t like that one ------------(A) either (C) neither

(B) too (D) alike

2. Sorry this is taking so long. Can you wait for --------------- ten minutes? (A) other (B) an other (C) another (D) few 3. Dave bought ------------- lamp for his mother‟s birthday. (A) a lovely new (B) a new lovely (C) a lovelier new (D) a new lovelier 4. Would you like ---------- tea? (A) anything (C) little

(B) some (D) few

5. Most ------------- work hard (A) of employees (C) employee

(B) employees (D) of employee

6. ------------- in this department was busy preparing for the conference last week. (A) Every person (B) Every people (C) All people (D) All the people 7. Don‟t forget to change all the oil ------------- 3,000 miles (A) each (B) every (C) another (D) other 8. Could you --------------- drinks to those guests who just arrived? (A) pour (B) serve (C) make (D) fill 9. Would you like another -------------- tea? (A) bowl (C) cup of

(B) cup (D) serving of

10. The holiday season is by far the busiest time of year for ------------- stores. (A) of the most (B) most (C) the most of (D) mostly 11. Jane couldn‟t find -------------- books on global warning in the bookstore. (A) no (B) some (C) any (D) almost 12. Some of the questions were --------------- that nobody could answer them. (A) so hard (B) too hard (C) very hard (D) not hard 47

13. -------------- all of prisoners were set free. (A) Most (C) Both

(B) Almost (D) Neither

14. I have never seen ------------- of the flowers you brought. (A) either (B) neither (C) none (D) some 15. He is very --------------person. (A) society (C) sociably

(B) sociable (D) socially

16. It‟s ------------ cold. (A) terror (C) terrible

(B) terrify (D) terribly

17. They have worked certainly -------------- to pass the test. (A) hard enough (B) hardly enough (C) enough hard (D) enough hardly 18. Is there ------------ about the project? (A) new anything (C) anything newly

(B) newly anything (D) anything new

19. ------------- three copies of this letter, please. (A) Be (B) Make

(B) Do (C) Build

20. If you violate any conditions specified therein, the contract will automatically --------(A) complete (B) annul (C) abolish (D) terminate 21. I‟d like to -------------- my check (A) cash (B) bill

(B) charge (D) exchange

22. Can I ------------- money online to Brazil? (A) refund (C) transfer

(B) reimburse (D) repay

23. With her friend‟s -------------- assistance, she completed her project successfully. (A) organ (B) organize (C) organization (D) organizational 24. Larry immediately said, “I can‟t go, nor -------------- want to go” (A) I (B) do (C) I do (D) do I 25. Mary, --------------- to catch up on her rest, went to bed early. (A) hope (B) hopes (C) hoped (D) hoping

48

26. I don‟t have --------------- on the hotels of the island. (A) much information (B) many informational (C) informs (D) many information 27. In the room, there were -------------- more books on biology than expected. (A) little (B) a little (C) much (D) many 28. I wish to remind all the students must ----------- in P.E. lesson. (A) be (B) have a role (C) take part (D) study 29. What ------------------ did that movie have on her? (A) advise (B) affect (C) effect (D) advice 30. They have ----------------- in Canada for two years. (A) is (B) be (C) were (D) been

49

Part 6: Incomplete Texts Questions 1 through 4 refer to the following note.

Hi Mary, I tried to call you, but there was no answer. I might be a bit late getting home tonight. I'll -------------- to cook dinner as I promised, but I won't have time to go to the supermarket. 1. (A) can (B) could (C) be able (D) would Can you pick up the following items on your way home? I'd really ---------- it. I need one 2. (A) appreciate (B) thanks (C) grateful (D) thank you kilogram of tomatoes, two onions, some garlic, two carrots, a kilogram of green beans, tomatoes, two onions, some garlic, two carrots, a kilogram of green beans, some parmesan cheese, and a packet of spaghetti. You can get a bottle of wine, too. Get whatever you like. I don‟t mind. Could you also get half a dozen eggs, a bag of flour (I think that we have run ---------------). I know we have sugar, so you don't need to buy that, and a 100g bar of dark 3. (A) (B) (C) (D)

off into out away

chocolate. Better make that 2 bars. Thanks. See you ---------------, 4. (A) lately (B) later (C) as of late (D) late Alex

50

Questions 5 through 8 refer to the following article.

Many parents worry that their children don‟t eat properly when they go off to university. While ---------- students live off instant noodles, they are not the average according to the 5. (A) all (B) some of (C) some (D) almost according to the results of a recent survey. Most of the university sophomores surveyed last month said that they cook for themselves at least three times a week. The other days they eat with friends. Many of the students said they take it ---------- to cook for each 6. (A) in turns (B) turns (C) in to turns (D) turning other. On the days that they don‟t cook, they said that they often eat cereal. This --------7. (A) is not (B) can not (C) is (D) has a bad thing. Most cereals these days are fortified with a wide range of vitamins and with a wide range of vitamins and minerals. Therefore, although it is not the most appetizing diet, provided they don‟t ------------ too many sugary cereals, it is not a completely unhealthy choice of foods. 8. (A) reject (B) chose (C) selected (D) choose

51

Questions 9 through 1 1 refer to the following article.

Memo To: All employees From: James Scott Re: Meeting changed As you know, we scheduled a meeting for Friday, June 22nd at 3:00 p.m. However, I feel that we need ----------- the time of this meeting because of re recent problems 9. (A) changed (B) to change (C) changing (D) change . I have talked to the manager, and he has agreed -----------10. (A) (B) (C) (D)

to move move moving of moving

it to Monday, June 18th. This will be an important meeting, so I ---------- see everyone 11. (A) wanting (B) wanting to (C) want to (D) want there. I apologize for causing any inconvenience.

52

Questions 12 through 15 refer to the following article.

Customer Relations, Supershopper Ltd., Mumford, West Cliffe Dear Sirs, Although I am sure you are used to receiving letters from dissatisfied -------------, I am 12. (A) users (B) consumers (C) visitors (D) customers writing to thank you for the kind treatment I received from to of your store ------------13. (A) users (B) employers (C) clerks (D) customers Earlier this week. I was doing my weekly shopping when I began to feel dizzy. It appears that I passed out. When I came to, a very nice young woman took me to the -----------14. (A) staff (B) members (C) employers (D) rest in the break room. She gave me a lot of water, and taking my shopping list, asked one of the packing staff ( a middle-aged gentleman) to gather up all the items on the list. This kind man then drove me home with the grocery items, which he had kindly picked up for me. I am most grateful to both these people. Please ------------- my thanks to them. 15. (A) assert (B) convey (C) converge (D) tell

53

Part 7: Reading Comprehension Question 1 through 4 refer to the following sign. King‟s Department Store Attention: Valued Customers We‟re moving to our new, more luxurious downtown location next month! To avoid the high costs of moving our products, we are having a special sale!

5th floor 4th floor 3rd floor 2nd floor 1st floor

All books and toys are 60% off. Furniture, tools, and hardware at up to 70% off. Women‟s and girl‟s clothing at 40% off (including swimwear). Men‟s and boy‟s clothing at 40% off (including swimwear). Up to 75% off all canned and dry goods in the supermarket.

1. Where can you find the sofa? (A) on the fourth floor (B) on the fifth floor (C) on the first floor (D) on the second floor 2. Where should Peter and his father go to buy new clothes? (A) to the fifth floor (B) to the fourth floor (C) to the third floor (D) to the second floor 3. Where should Jack go to buy a new dictionary for English class? (A) to the fifth floor (B) to the fourth floor (C) to the third floor (D) to the second floor 4. Why is the store having a special sale? (A) The floors were rearranged. (B) The store will be relocating. (C) The store rooms are full. (D) It is closing down.

54

Questions 5 through 7 are based on the following letter.

Limited Car Manufacturing Company 1 Highway Circle Harristown, Indiana To: Stephen Jenson, Comfort Ride Seating company Dear Mr. Jenson: It has come to our attention that your production of car seats for our latest car has decreased. As we previously discussed in our agreement, your company needs to produce 1,000 s e a t s per month in order for us to stay on schedule. Eric White, the production director, was also informed o f this demand. Currently, only 800 seats per month a r e bein g manufactured. This decrease in production is causing some serious p rob lem s f o r our company. We must talk about this issue immediately. If your company cannot meet the production quantity specified, we will be forced t o look for another company. Please contact us as soon a s possible. Sincerely, Henderson Production Manager

5. Which of the following best describes the nature of this letter? (A) Congratulatory (B) Pessimistic (C) Jovial (D) Complaining 6. By how much has the production been reduced? (A) 200 seats (B) 800 seats (C) 1,000 seats (D) 1,800 seats 7. Who wrote this letter? (A) Eric White (B) Henderson (C) Mr. Jenson (D) The production director

55

Questions 8 through 10 are based on the following article.

Jason Avery has been a professional comedian for over fifteen years. As he says, "It‟s not easy to make people la u gh every night. As a comedian, you have to keep inventing new jokes all the time to keep the public happy." Obviously, Mr. Avery has been very successful doing t h is . He has appeared on many popular television shows and has been a guest of the President at special ceremonies. Avery's most recent works, in addition to television shows, have been several popular movies and cable broadcast comedy programs. He a ls o frequently donates his time to charity events to help raise money for children in need. The most rewarding part of his job, Avery says he likes to see people smile. As Avery says, "It's a hard job, but someone has to do it!"

8. What is Avery's occupation? (A) Actor (B) Broadcaster (C) Comedian (D) Donor 9. What does Avery like best about his job? (A) Helping others (B) Seeing people smile (C) Being famous (D) Meeting the President 10. Who benefits from Avery's charity performances? (A) Politicians (B) Senior citizens (C) Acting students (D) Children

56

Questions 11 through 15 are based on the following letters. To: Holiday Hotel ([email protected]) From: Morris Knowles ([email protected]) Subject: Check-out policy Date: November 4 2006 Dear sir, I recently made an online reservation, but I have just realized that I do not know your check-out policy. I would like to leave around lunchtime. In fact, I plan to have a lunchtime business meeting in the hotel restaurant before leaving. I really do not want to carry my bags with me during the meeting. Is it possible to check it: It‟s around 2:00 pm? Sincerely, Morris Knowles

To: Morris Knowles ([email protected]) From: Holiday Hotel ([email protected]) Subject: Re: check-out policy Date: November 5 2006 Dear Mr. Knowles, Here is a copy of our check-out policy Holiday Hotel Check-out Procedures: Thank you for staying in Holiday Hotel. We hope you have enjoyed your stay. Please note that check-out time is 11:00 a.m. If you have pre-paid for your room, simply leave your key on the dresser and have a nice day. If you have not pre-paid, you may pay the receptionist by check or credit card in the hotel lobby if you need assistance, you may dial the hotel secretary on extension 611. I am afraid you will need to leave your room by our check-out time of 11:00 a.m. However, we would be happy to store your luggage at the front desk while you conduct your meeting.

11. What time does Mr. Knowles hope to check out? (A) 11: 00 a.m (C) 12:00 p.m

(B) 11: 00 p.m (D) 2:00 p.m

12. What will Mr. Knowles do before leaving the hotel? (A) Make an online reservation (B) Have a business meeting (C) Make an important phone call (D) Apply for a job 13. Where can clients pay for their stay? (A) In the room (C) On the phone

(B) In the lobby (D) On the dresser

14. Who can a client call for assistance? (A) The bellhop (C) The secretary

(B) The receptionist (D) The manager

15. What should Mr. Knowles do with his luggage? (A) Leave it in the room (C) Take it to the restaurant

(B) Leave it at the front desk (D) Leave it in the lobby

57

Practice Test 1 Part 1: Picture Description Listen and choose the statement that best describe the picture

(A) (B) (C) (D)

1

2.

(A) (B) (C) (D)

(A) (B) (C) (D)

3.

58

4. (A) (B) (C) (D)

5.

(A) (B) (C) (D)

Part 2: Questions and Responses Listen and choose the correct answer 1.

(A)

(B)

(C)

2.

(A)

(B)

(C)

3.

(A)

(B)

(C)

4.

(A)

(B)

(C)

5.

(A)

(B)

(C)

59

Part 3: Short Conversations Listen and choose the best answer to each question. 1. What are the man and woman doing? (A) Parking a car (B) Buying a car (C) Looking for a friend (D) Talking about space 2. Why is the man annoyed? (A) They wasted a lot of time (B) He forgot to wear a watch (C) The woman always wastes his time (D) He hates parking 3. What did it take them fifteen minutes to do? (A) Find the car (B) Get ready (C) Get downtown (D) Find a parking spot

4. Why is the woman angry? (A) She is always angry. (B) She was late for an appointment. (C) The man was late for their appointment. (D) She lost her handbag. 5. What time is it now probably? (A) 1:00 p.m. (B) 2:00 p.m. (C) 2:30 p.m. (D) 3:30 p.m. 6. Which sentence is probably true? (A) The man is often late. (B) The man is never late. (C) The man has a car. (D) The woman took the bus.

60

Part 4: Short Talks Listen and choose the best answer to each question. 1. If there is a fire, what should you do? (A) Run out of the building (B) Stay seated (C) Walk out of the building (D) Jog out of the building 2. When outside, where should you wait? (A) On the basketball court (B) In the parking lot (C) Beside the school (D) On the football field 3.

How often will the fire drills be held? (A) Three times a year (B) Four times a year (C) Twice a year (D) Once each year

4. When should the students begin the test? (A) When they are given the test paper (B) When the teacher says paper (C) When they hear the bell (D) When they want 5. What should the students write on the first page? (A) Their teacher‟s name (B) Their age (C) Their phone number (D) Their name and class number 6. How much time do the students have to finish the test? (A) One hour (B) Two hours (C) Three hours (D) Five hours

61

Part 5: Complete sentences Choose the word or phrase that best completes the sentence. 1. Those are ------------- books (A) her (C) she‟s

(B) hers (D) she

2. I‟m --------- sick (A) feel (C) feeling

(B) feels (D) felt

3. ------------- going to the soccer game (A) Their (C) They‟re

(B) There (D) There‟re

4. I have ----------- friends. (A) much (C) any

(B) many (D) most

5. Would you like something -------------? (A) drink (C) to drink

(B) drinks (D) to drinks

6. Can I borrow ------------- money? (A) a (C) most

(B) some (D) many

7. I don‟t have -------------- money. (A) any (C) an

(B) a (D) many

8. She has ------------ eyes. (A) beauty (C) a beauty

(B) beautiful (D) a beautiful

9. My mother ---------- in a bank. (A) work (C) working

(B) works (D) worker

10. I‟ve been learning English ------------- three years. (A) for (B) since (C) while (D) in 11. Where ----------- he work? (A) is (C) do

(B) are (D) does

12. Honey is made ---------- bees. (A) from (C) by

(B) to (D) out of 62

13. --------------- is fun. (A) Swimming (C) Swim

(B) A swimming (D) Be swimming

14. What are you ----------- next weekend? (A) do (C) did

(B) doing (D) done

15. Do you have ----------- computer at home? (A) a (C) some

(B) an (D) those

16. I usually ---------- the bus to school. (A) take (C) take on

(B) takes (D) takes on

17. We --------- to go shopping. (A) enjoy (C) need

(B) must (D) couldn‟t

18. He works ---------- a doctor in that hospital. (A) at (C) and

(B) as (D) any

19. We enjoy ----------- in the mountain. (A) hike (C) hiking

(B) to hike (D) to hiking

20. She‟s waiting ------------- her friend (A) to (C) about

(B) for (D) because

21. I have no money. I‟m ---------(A) broke (C) breaking

(B) broken (D) break

22. That movie was very ---------(A) interest (C) interesting

(B) interested (D) to interest

23. We‟ve lived here ----------- ten years. (A) since (C) from

(B) during (D) for

24. She is the teacher ----------- helped me. (A) who (C) whose

(B) when (D) what

25. They hope ---------- to Singapore next year. (A) travel (C) traveling

(B) travels (D) to travel

63

26. ------------ my friends take music lessons. (A) Any (C) Any of

(B) Many (D) Many of

27. We don‟t have ------------- problems. (A) a (C) no

(B) any (D) none

28. They have not finished the project -----------(A) already (C) yet

(B) soon (D) before

29. Sue always ----------- a walk in the morning. (A) takes (C) goes

(B) makes (D) does

30. The wind is blowing ---------- today. (A) difficult (C) difficulty

(B) hard (D) hardly

64

Part 6: Complete texts Questions 31 through 33 refer to the following note.

Dear Sally, I am going to ------------- grandmother‟s house this afternoon. I will be out when you get home 31. (A) the (B) our (C) your (D) yours from school, so you will need to get your own dinner. Help yourself ----------- anything in the 32. (A) with (B) to (C) by (D) over refrigerator. There is some cheese and plenty of vegetables. There is bread, too. You can pick ---------- apples from the tree in the garden if you want. 33. (A) any (B) some (C) all (D) almost As a special treat, I have bought some chocolate fudge ice cream. Don‟t eat too much – I want to try some, too! Do your homework, OK? I‟ll be home about eight o‟clock. See you later. Mom

65

Questions 34 through 36 refer to the following memo.

MEMO To: Sales Staff From: Marketing Re: Christmas sales target Date: October 15th It is time to start thinking -------------- our Christmas sales. Last year, we saw very low 34. (A) on (B) about (C) for (D) off profits, so we really need to improve things this year. The store wants to double its profits, so we have a lot of work to do. We will have a meeting next Tuesday a t three o‟clock to discuss this. Please think of some good ideas to increase sales. If we reach the new target, ------------ will get a bonus. If we are not successful, there will be no bonuses. 35. (A) everyone (B) no one (C) anyone (D) each The store manager will come to the meeting. He is going to --------------- us his ideas and 36. (A) say (B) talk (C) tell (D) speak listen to our opinions. This is a very important meeting. Don‟t miss it.

66

Questions 37 through 39 refer to the following advertisement .

Getaway Tours We now have the details of our Summer Specials. As always we have some real treats. Just take a look a sample of ------------- we have on offer this year: six nights in Guam, 37. (A) that (B) what (C) which (D) why staying in a luxury hotel with breakfast included- starts at just $50 per person per night. Five nights in Hong Kong, staying in a five star hotel, including a guided harbor boat cruise – starts as just $75 per person per night. ------------ of our prices include travel 38. (A) Almost (B) All (C) Nearly all (D) At all insurance and free transportation from the airport ---------- to your hotel. Give us a call at 39. (A) straightly (B) directly (C) firstly (D) immediate 234-0009 for reservations or further information.

67

Questions 40 through 42 refer to the following email .

To: Nina Tendo From: Jane Sims Subject: Customer Complaint#00234 Date: May 25th Dear Ms. Tendo, Thank you for your email last week. I am very sorry to hear that you had a problem with a United Foods product. You ------------ us that you found ants in a packet of Jolly Cowboy 40. (A) identified (B) informed (C) information (D) identity cookies. We went to the store where you bought the cookies. We found ants in se veral packets of Jolly Cowboy cookies. We found that ants had entered the store room. We treated the problem, and there are -------------- ants in the store room. We are very sorry that 41. (A) not more (B) more (C) no more (D) any more this happened, and hope you will continue to buy our food products. I am sending you a $20 coupon, -----------, you can print out and use in any store to buy United Foods products. 42. (A) which (B) what (C) where (D) who Sincerely, Jane Sims Customer Service Manager

68

Questions 43 through 45 refer to the following message.

Lucy, Can you ask the cleaner to empty all the trashcans before she goes home every night? ------- every morning this week. I have found the trash can in my office full of trash. 43. (A) Most (B) At most (C) Almost all (D) Almost Also, she should check that there is ----------- toilet paper in the rest rooms. Last week 44. (A) enough (B) such (C) each of (D) an we ran out of paper everyday. It is inconvenient and --------- keeps complaining to me. 45. (A) everyone (B) no one (C) all (D) anyone Thanks for your help, Carrie

69

Part 7: Reading comprehension Question 46 through 47 refer to the following sign.

CAUTION: DETOUR Drive slowly Due to the recent cold and snow resulting in dangerous driving conditions on the Bukowski Bridge, Route 66 is now closed to traffic. All traffic going south on Route 66 must now travel East on Route 75 to Williamsburg and then turn south on Route 19. Remember: Speed skills, so drive safely! Happy holidays and thank you for your cooperation, State Highway Department

46. Where would you probably see this sign? (A) On a beach (B) On a building (C) In a forest (D) Next to a highway

47. When would this sign probably be posted? (A) In a rain storm (B) In summer (C) In winter (D) On Saturday

70

Quetions 48 through 5 0 are based on the following advertisement.

Always tired? No energy? Do you read a little extra strength to get through that tough workday? If you answered “yes”, then try Vitagetic, the new energy drink. Vitagetic has all the vitamins that your body needs to maintain a high level of energy all day long. It is an easy way to get all your vitamins. You don’t have to take lots of vitamin pills. It comes in three fantastic flavors: Lemon, Orange, and Apple, and tastes great. Have one bottle at breakfast and you will feel healthy and energetic until quitting time. You can find it at all good supermarkets.

48. Who should drink Vitagetic? (A) People who are often tired. (B) People who have a lot of energy. (C) People who are bored. (D) People who can‟t sleep.

49. Why is Vitagetic good for you? (A) Because it has oranges and apples. (B) Because it has vitamins. (C) Because it comes in a bottle. (D) Because you drink it at breakfast.

50. Which of the following is NOT true? (A) You have to take vitamin pills with Vitagetic. (B) You can buy Vitagetic at supermarkets. (C) Vitagetic is a good breakfast drink. (D) Vitagetic is delicious.

71

Questions 5 1 through 53 are based on the following sign.

Recycling Instructions Please separate your trash into the following bins. Glass and plastic go in the large blue bin. Paper and cardboard products go in the yellow bin. Aluminum cans go in the purple bin, but all other metals go in the red bin. Food waste should be put in the green bin. Thank you for your cooperation, Scherk Property Management

51. Where should you put cola bottles? (A) In the blue bin (B) In the yellow bin (C) In the red bin (D) In the green bin

52. Which of the following would you put in the yellow bin? (A) Old shoes (B) Food (C) Magazines (D) A broken TV

53. Which of the following can NOT be put in the green bin? (A) Newspapers (B) A leftover sandwich (C) Orange peel (D) Rice

74

Questions 5 4 through 56 are based on the following sign.

JADE LION Szechuan Style Chinese Restaurant Dimsum, Peking Duck, Ginger Beef, and all your mouth-watering favorites Try our $9.99 lunch buffet Monday through Friday, 11:00 a.m – 2:00 p.m. Planning a party? No problem, book one of our many rooms. 56 West Sycamore Street (beside Eaton‟s Department Store) Call 476-5593 for group reservations Call 476-5594 for takeout orders (pick-up or delivery) Hours: Monday- Friday 11:00 a.m – 10:00 p.m. Saturday 12:00 p.m. – 11:00 p.m. No MSG available upon request

54. What kind of food does the Jade Lion Restaurant serve? (A) Korean food (B) American food (C) French food (D) Chinese food 55. Which day is the restaurant is NOT open? (A) Monday (B) Tuesday (C) Saturday (D) Sunday 56. What is the address of the restaurant? (A) Jade Lion (B) 11:00 a.m – 9:00 p.m. (C) 56 West Sycamore Street (D) Beside Eaton‟s

75

Questions 5 7 through 60 are based on the following l e t t e r .

April 10th. Dear Harry, Our new house is so nice. We have been living here for two months. We have such a big living room and dining room. Upstairs, there are three bedrooms and two bathrooms. Behind the house, we have trees, a swimming pool, a garden for growing vegetables. In the front, we have a beautiful flower garden. I hope that you will come to visit someday. Best wishes, Jack

57. What is Jack writing about? (A) His new house (B) His family (C) His dog (D) His friend

58. What is upstairs? (A) The living room (B) The kitchen (C) The swimming pool (D) Three bedrooms

59. What is in front of the house? (A) A vegetable garden (B) A flower garden (C) A swimming pool (D) Three bathrooms

60. When did Jack move to his new house? (A) In two months (B) Recently (C) In April (D) In February

76

Practice Test 2 Part 1: Picture Description Listen and choose the statement that best describe the picture

(A) (B) (C) (D)

1.

2.

(A) (B) (C) (D)

(A) (B) (C) (D) 3.

77

4. (A) (B) (C) (D)

5. (A) (B) (C) (D)

Part 2: Questions and Responses Listen and choose the correct answer 1.

(A)

(B)

(C)

2.

(A)

(B)

(C)

3.

(A)

(B)

(C)

4.

(A)

(B)

(C)

5.

(A)

(B)

(C)

78

Part 3: Short Conversations Listen and choose the best answer to each question. 1. Where does the man want to go? (A) To the bookstore (B) To the bank (C) To the movies (D) To the library 2. How can he get to the bank? (A) Turn left, then go straight (B) Go straight for two blocks, then turn left (C) Go straight for one block, then turn left (D) Go straight for one block then turn right 3. What does the woman say about the bookstore? (A) That it has a wide variety of books (B) That it is difficult to find (C) That it is quite far away (D) That the man might not find the book he needs

4. What is the man going to do after the conversation? (A) Go home (B) Take a bath (C) Go to the supermarket (D) Clean the bathroom 5. What does the woman want him to buy? (A) Some towels (B) A bath (C) Some shampoo (D) Some soap 6. What is the man worried about? (A) The supermarket will be too crowded (B) He might buy the wrong soap. (C) The woman will ask him to buy too many things. (D) He thinks he might run out of money.

79

Part 4: Short Talks Listen and choose the best answer to each question. 1. How long has the speaker been studying judo? (A) Since he was ten (B) Before he was ten (C) Over ten years (D) After ten years 2. What happened to the speaker at his last competition? (A) He lost his match. (B) His opponent hurt his shoulder. (C) The referee studied judo. (D) He hurt his opponent‟s shoulder. 3. What did the speaker do to his opponent? (A) He punched him to the ground. (B) He kicked him to the ground. (C) He threw him to the ground. (D) He hurt his opponent‟s hand.

4. What does the speaker NOT have experience doing? (A) Cooking (B) Sweeping (C) Serving (D) Watering 5. When can the speaker start the new job? (A) Yesterday (B) A few years (C) This morning (D) Tomorrow 6. What does the speaker want? (A) Lunch in a restaurant (B) A new job (C) Some plants (D) Customers

80

Part 5: Incomplete sentences Choose the word or phrase that best completes the sentence. 1 --------- boarding the aircraft, passengers with small children and passengers with disabilities will be given priority. (A) During (B) Due to (C) When (D) Because 2. A plethora of study programs now available online has made it possible for working people to gain certificates or degree far ----------- than ever before. (A) more easily (B) easier (C) easiest (D) easily 3. Ms Jenifer has been a --------------- colleague of mine for seven years and it has been pleasure to work with her. (A) close (B) closely (C) closing (D) closure 4. A career choice is ------------ one has to decide for oneself. (A) everything (B) nothing (C) something (D) anything 5. Our customers expect good service from us and we should do our best to live -------- their expectations. (A) in for (B) on for (C) up for (D) up to 6. With the newly built manufacturing facility in North Korea, the company will be able to produce -------- as many units as before. (A) two (B) double (C) twice (D) second 7. During job interview, applicants should try to remain as calm as they can -----------nervous they may actually be. (A) even if (B) however (C) although (D) nonetheless 8. The local business establishments ------------ dry cleaners and restaurants are reporting more incidents of thefts and burglaries in recent months. (A) such (B) such a (C) such as (D) such that 9. She encouraged him to speak ---------- by enunciating words carefully when she spoke to him. (A) clearness (B) clearly (C) clearing (D) cleared

81

10. Please visit the student center for the latest update on the up-to-date program of ----------(A) activeness (B) actively (C) active (D) activities 11. The best way to keep parents ---------- is to keep them informed and for their children to thrive. (A) satisfaction (B) satisfy (C) satisfyingly (D) satisfied 12. She develops her painting by-------------, rather than receiving a formal art education. (A) he (B) her (C) herself (D) hers 13. The forum encouraged the guests to provide ----------- criticism of each work presented and reflect on their own experience. (A) constructor (B) construction (C) constructive (D) constructs 14. The now filing system proposed by Anders Consulting already ------------- efficiency by thirty percent since it was implemented. (A) has increased (B) increased (C) will increase (D) was increasing 15. When she told me that I got a job, I was so excited that I did not know what ---------------(A) to tell (B) to talk (C) to say (D) to speak 16. -------------- a photographer is one of the most exciting jobs in the world. (A) Doing (B) Seeing (C) Being (D) Making 17. I call her on at least three occasions, but she ------------ got back to me. (A) rarely (B) sometimes (C) never (D) usually 18. The committee ----------- that we do not pursue legal action at this time, but wait for the result of the tests. (A) suggesting (B) is recommending (C) has recommended (D) is recommended 19. ------------- was always one of my strong points during my M.B.A. course. (A) To advertise (B) Advertising (C) Advertisement (D) To advertising 20. You will find all the information you need ------------- this computer disk. (A) on (B) in (C) at (D) by

82

21. He tries hard, but I ----------- don‟t think he is doing a good job. (A) very (B) really (C) almost (D) exceptionally 22. My uncle works as a cook ------------- prison, but my aunt wants him to work as a chef in the hotel. (A) in (B) in a (C) in the (D) in some 23. I would ------------- coffee to tea. (A) choose (C) prefer

(B) like (D) care

24. The new computer system designed for the post office -------------- accounts to be managed over the Internet. (A) are allowing (B) allow (C) will allow (D) will be going to allow 25. She really ------------ to learn to control her temper in the office. (A) has (B) have (C) hasn‟t (D) having 26. ----------- people who love their work needed time to relax and enjoy themselves. (A) Also (B) Although (C) Even (D) Despite 27. A recent study of corporate managers -------------- most executives do not find their work satisfying. (A) found that (B) which discovered (C) revealing (D) founded 28. If you do not read the business page regularly, you ----------- to miss reports of the important business transaction. (A) are going (B) will (C) would be going (D) had 29. The new overtime regulations will take -------------- at the start of next month. (A) affect (B) effect (B) efficiency (C) affectations 30. “What ------------- when you saw the man?” the police officer asked. (A) have you done (B) are you doing (C) were you doing (D) do you do

83

Part 6: Incomplete Texts Questions 3 1 through 34 are based on the following message To: Fiona Kim From: Baljit Singh Subject: Urgent Business Ms Kim: I wish you to present yourself at my office at 9:00 a.m. tomorrow. It has been -------- to my 31. (A) made (B) presented (C) brought (D) held Attention. By various members of staff that you have not been acting in a very cooperative manner of late, and I feel we need to meet to discuss the ----------. I was very surprised to 32. (A) circumstance (B) situation (C) development (D) event hear this, and although I have been ignoring it, hoping that this was a temporary matter, it has been going on long enough that I think we need to talk. I will not go into -------- in this memo, but needless to say we will be discussing three major 33. (A) details (B) information (C) carefully (D) in detail incidents which have caused us to lose important contracts. I have not yet decided what will happen to your position at this company; It depends rather on the outcome of our meeting tomorrow and the outcome of the second meeting that I will have with both of your --------- tomorrow afternoon. 34. (A) watchers (B) foreman (C) leaders (D) supervisors

84

Questions 35 through 37 are based on the following email.

To: [email protected] From: homefurnishings@ homecare.com Date: Jan 15 2006 Subject: Order confirmation #26678

Dear Mr. Lyons, I am -------this email to confirm your recent order with Home Furnishings. Your order 35. (A) send (B) to send (C) sending (D) sent has been processed, and your items will be sent on January 17'h. The items should arrive by January 20'h. Please let us know if you would prefer --------- these items 36. (A) delivering (B) to have (C) having (D) had delivered in the afternoon. Thank you for ------------- Home Furnishings. We hope to 37. (A) realizing (B) seeing (C) knowing (D) using serve you again soon. Sincerely, Brenda Smith Sales Executive

85

Questions 38 through 41 are based on the following notice.

Volunteers Wanted The Psychology Department of Queenstown University is ----------- volunteers to take part 38. (A) searching (B) looking for (C) necessary (D) investigating in an experiment. We need 50 volunteers: 75 men and 75 women. Volunteers should be aged -------------- 20 and 25 and should not be taking any kind of medication. Volunteers 39. (A) under (B) by (C) between (D) from must not be enrolled in any of the university‟s current psychology courses. The experiment will take between thirty to forty minutes to complete. Participants will be asked to answer a series of questions after ------------a short film. A small fee will be paid to participants. 40. (A) have watched (B) watch (C) watching (D) to watch. Coffee and tea will be also served during the film. If you are ---------, please contact the Department secretary on 990-887 before January 30th. 41. (A) interested (B) interesting (C) have an interest in (D) interest

86

Questions 42 through 45 are based on the following article

Over the last few months, we have received ---------- letters from readers, asking us where 42. (A) much (B) a lot of (C) number of (D) lots they can buy some of the items that have been on display in the background of a number of our fashion features. It almost seems that the furnishings and decorative items from those photo shoots have---------------------- interest than the clothes themselves. 43. (A) aroused more (B) aroused (C) provoked (D) made more . For this reason you will notice that, starting on page 78 of this month‟s edition, we have introduced a new feature entitled “Where can I buy….?” This new section will appear immediately after our main fashion photo feature each month. We will be listing the designers and manufacturers for just about everything you can see in each photo, and we will be giving you the numbers and websites you need to find out more. Many of these items will be made available at a ---------- to magazine subscribers. 44. (A) bargain (B) price off (C) major retailer (D) discount All the more reason to take out the subscription! Take a ----------- and let us know what you think. 45. (A) break (B) time (C) off (D) look

.

87

Part 7: Reading comprehension Questions 46 through 48 are based on the following notice

ANNOUNCEMENT: TIME CHANGE Tuesday‟s Movie Club meeting at the Megaplex cinema will be moved to Friday at 3:00 p.m. We will be watching the movie The Winzard of Oz. It is the 20th time we have shown it. During the movie, we will serve popcorn and drinks at our usual low prices. Remember, as part of our special offer, this week you can bring one friend for free. We have seating for only 300 people. This is one of our most popular movies, so don‟t be late. We hope to see you soon.

46. Which of the following is true? (A) The meeting will be on Tuesday. (B) The meeting will be moved from Friday to Tuesday (C) The meeting will be on Friday (D) The meeting will be moved from Thursday to Friday. 47. What is the special offer? (A) Free popcorn (B) Free drinks (C) A free hotdog (D) A free ticket for a friend 48. How many people can watch the movie? (A) Three (B) Twenty (C) 300 (D) Everyone

88

Questions 4 9 through 52 are based on the following m e mo r a n d u m .

MEMORANDUM To: All employees From: Sidney Flanders, Office Manager Date: September 4 Re: Vacation Days Since it is never too early to be prepared, I am requesting that all employees promptly submit to their departmental managers any requests for vacation days for the remainder of the year. As you know, the end of the year is our busiest time, and there is always the problem of having to schedule vacations and personal time off around the end-of-year holidays. This is the most popular time for vacations, apart from the summer months, and it takes a lot of time to coordinate. I will work with all department managers to see that all vacation requests are honored where possible, except in cases where it is anticipated that the temporary staff will not be able to cover the projected workload. Unfortunately, we will have to request that some employees (generally employees with little seniority) postpone their vacation plans until a more convenient time. Those of you who do not get your first choice of vacation this time round will be given priority next time. However, please note that this is still not flat guarantee that you will get your choice, even then. I would like to ask that all requests be made to the appropriate offices before the end of the week. Thank you for your cooperation in this matter. Rest assured that I will do very best to accommodate all wishes.

49. Who will be receiving vacation requests? (A) The personal secretary (B) The office manager (C) All employees (D) Temporary employees 50. What is true for employees with more seniority? (A) They will not need to make vacation requests. (B) They may be asked to postpone their vacations (C) They will probably have their vacation request honored. (D) They will not receive this memorandum. 51. According to the memo, when should requests for vacation time be made? (A) By the end of the day (B) By the end of the week (C) By the end of the month (D) By the end of the year 52. According to the memo, what problem often occurs? (A) Employees do not schedule enough vacation time. (B) Vacation time is used too early in the year. (C) Departmental managers often reject request for time off. (D) There is difficulty scheduling vacations at year‟s end. 89

Questions 5 3 through 56 are based on the following advertisement.

Worried that you or someone in your family is at risk of hurting the other members of your family? Try the People‟s Community Clinic – we have over 20 years of experience assisting people just like you. The People‟s Community Clinic is a public service organization open to all members of the local community who fell they and their family are in need of assistance in dealing with substance abuse. Mediated, anti-violence workshops for family members of substance abusers are available at your local People‟s Community Clinic at regular scheduled times, or by appointment. Schedules vary from site to site, so please call before you come in. It has been our experience that the workshops are most helpful when all family members are able to attend, but don‟t feel that you cannot attend alone: Sometimes it helps just knowing that you are not the only person in your situation. Group sessions are free and open to the public. Private counseling is available by appointment only. All applicants for private counseling must submit to a screening interview. Private counseling fees are calculated on a sliding scale, based on your ability to pay. If your family is at risk, don‟t wait until the violence begins. It is not shameful to seek help. Our fully-qualified counselors are professionals who will listen and advise you without judgment. Call or come in today. People‟s Community Clinic We‟re here for you.

53. What is the cost of group sessions? 54. How does one arrange for a private session? (A) Fees are based on a sliding scale. (A) Just come in (B) They are free. (B) Come with your family (C) It is based on your ability to pay. (C) Make an appointment (D) The advertisement doesn‟t say. (D) Get a doctor‟s approval 55. Who are the workshops intended for? (A) Substance abusers (B) Families of substances abusers (C) Violent families (D) Criminals 56. How is the cost of private counseling sessions decided? (A) They charge according to the number of people attending. (B) It depends upon the family‟s disposable income. (C) It is up to the family members to decide how much they want to pay. (D) Families pay according to how long the session lasts.

90

Questions 5 7 through 60 are based on the following article.

Saving Time When Computing Here is the second installment in our monthly “Computing for Business Users” guides. A common complain among computer users is the amount of time they spend waiting for their laptop to start up and perform its tasks. Here are some handy tips on how to eliminate some of that waiting time.  Don‟t shut your laptop all the way off between meetings; just put it in the lowbattery consumption “Standby” mode. Shutting it down and waiting for it to reboot at the next meetings wastes valuable time. This way you are ready to start as soon as you arrive.  Having too many programs in your computer‟s Start Up folder really shows things down. The solution? Eliminate all unnecessary programs.  Stop too many programs from running at the same time when you start up the computer. Don‟t know how? It‟s easy if you follow these instructions: click the Start menu, choose Run, then type “msconfig” to launch the System Configuration Utility. Next, on the “General” tab, click “Selective Startup”. Then go to the “Startup” tab and uncheck any startup items that aren‟t necessary. These are small but simple changes that you can make yourself to save precious time. Next issue we‟ll be looking at extending your battery life.

57. Who is this article intended for? (A) Computer programmers (B) Business people (C) Program designers (D) Professional computer game players

59. How often are these computer articles published? (A) Every day (B) Every week (C) Once a month (D) Twice a year

58. What does this article explain? (A) How to start up your computer (B) How to improve the speed of computer (C) How to change a battery (D) Where to buy the best computer

60. What is the topic of the next article? (A) Buying a new computer (B) Making your battery last longer (C) Computer accessories (D) Security

91

Practice Test 3 Part 1: Picture Description Listen and choose the statement that best describe the picture

(A) (B) (C) (D)

1

2 (A) (B) (C) (D)

(A) (B) (C) (D)

3

92

4 (A) (B) (C) (D)

5

(A) (B) (C) (D)

Part 2: Questions and Responses Listen and choose the correct answer 1.

(A)

(B)

(C)

2.

(A)

(B)

(C)

3.

(A)

(B)

(C)

4.

(A)

(B)

(C)

5.

(A)

(B)

(C)

93

Part 3: Short Conversations Listen and choose the best answer to each question. 1. How much is a large envelope? (A) 75 cents (B) 50 cents (C) 150 cents (D) 10 cents 2. How many different types of envelopes are available? (A) Three (B) Four (C) Five (D) Six 3. How much would 100 small envelopes cost? (A) $45 (B) $50 (C) $70 (D) $75

4. Where is this conversation taking place? (A) In an office (B) At a sports center (C) At a railway station (D) In a department store 5. Where does the woman want to go? (A) To the shopping mall (B) To the bank (C) To her mother‟s house (D) To Midland 6. Which train will the woman take? (A) The express because it is cheaper (B) Neither, she will walk home. (C) Both of them (D) The local, because it is cheaper.

94

Part 4: Short Talks Listen and choose the best answer to each question. 1. Who do you think this ad is for? (A) People who have lots of energy (B) People who don‟t have much work (C) People who find it difficult to relax (D) People who are relaxed 2. How does yoga help people who work long days? (A) It gives them more energy. (B) It gives them a new job. (C) It doesn‟t help them. (D) It makes them stressed. 3. What days do the phones lines stay open? (A) Every day (B) Monday until Sunday (C) Every day except Tuesday (D) Every day except Monday

4. What did the speaker think of Canada? (A) She loved it. (B) She didn‟t like it. (C) She doesn‟t want to go back. (D) She hated it. 5. Who did the speaker walk in the mountains with? (A) Nobody (B) Her friends (C) Canada (D) Vacation 6. When did the speaker meet her friends for dinner? (A) In the mornings (B) Before her walks (C) After her walks (D) Two nights a week.

95

Part 5: Complete sentences Choose the word or phrase that best completes the sentence. 1. The flight arrived late ------------- stormy weather (B) for (C) then after

(B) because of (D) instead of

2. Last month the manager --------------- to the president that he hire a specialist to run a communication network (A) suggested (B) suggest (C) suggesting (D) suggestion 3. On Tuesday, the National Association of Realtor will release ------------ report on home sales for October. (A) her (B) its (C) his (D) an 4. Company profits reached $6 billion in September, the ---------- of the decade. (A) widest (B) highest (C) least (D) deepest 5. Bookstore chains now control ------------ 4% of the market for hardcover trade books. (A) to

(B) on

(C) about

(D) in

6. We are a business strategy consulting firm ------------- the top management of leading corporations. (A) are serving (B) served

(B) serve (D) serving

7. Special reduced postal rates ------------- for multiple copies. (A) are available (C) available

(B) availability (C) availed

8. Overall, vacations in the US are expected to -------------- a modest 3% more this year, compared to last. (A) cost (B) costing (B) costs (D) will cost 9. This year, for the first --------, all new model cars are required to have front seat airbags. (A) event (B) reason (C) place (D) time 10. The holiday season is by far the busiest time of year for ------------- stores. (A) of the most (B) most (C) the mostly (D) mostly

96

11. There will be a meeting for anyone interested in learning about the new building plans at 5 p.m -------------(A) on next Friday (B) at next Friday (C) in next Friday (D) next Friday 12. The large conference room is located ---------------the third floor, not the second as stated in the brochure. (A) on (B) by (C) to (D) within 13. ------------ to the latest figures, our competitors are not doing as well as they would like us to think. (A) Averaging (B) Abiding (C) According (D) Actually 14. It never ceases to amaze me just ------------ job applicants can‟t even put together a basic resume and cover letter. (A) how many (B) how to (C) how about (D) how 15. Although he was not to blame, he -------------- responsibility for the production mistakes. (A) held (B) took (C) gave (D) made 16. I don‟t think I‟ve had a more difficult time ---------- here than I did during the events of the past few days. (A) to work (B) working (C) at work (D) worked 17. Five years ------------, the Lewis sisters ran the most successful interior design business in Los Angeles. (A) ago (B) then (C) since (D) after 18. The new computer system designed for the post office ------------ accounts to be managed over the Internet. (A) are allowing (B) allow (C) will allow (D) will be going to allow 19. They ------------- to build an addition to their hotel to accommodate many guests. (A) must (B) ought (C) should (D) have to 20. The company thought the shipment ---------- in time for the holiday shipping season. (A) can arrive (B) is arriving (C) will arrive (D) would arrive 21. ----------- the unsuccessful summer season, the manager wants to go ahead with expansion plans (A) Because (B) Although (C) Due (D) Despite 97

22. If you have an advanced degree, there are ---------- more job opportunities available to you. (A) seldom (B) many (C) few (D) too many 23. Look ------------- these statistics and tell me if you think they are accurate. (A) over (B) from (C) to (D) under 24. I have noticed that when the boss is away, employees like to relax and enjoy----------(A) myself (B) ourselves (C) himself (D) themselves 25. She hasn‟t finished the monthly financial reports -------------, but says she should have them by the end of the day. (A) by this time (B) still (C) yet (D) then 26. He has to finish the meeting and get to the airport ……….. 6 p.m. (A) to (B) on (C) until (D) by 27. For professionals at the beginning of their careers, leisure time is a ------------- commodity. (A) high-priced (B) regular (C) common (D) scarce 28. By the end of the day, you must decide ------------ or not you want to attend the conference next month. (A) whether (B) if (C) however (D) rather 29. In a memo to all department managers, the employees -------------- more flexible working hours. (A) inquired (B) asked (C) said (D) requested 30. Our competitor orders a high ------------ of its equipment from foreign companies. (A) percentage (B) range (C) level (D) degree

98

Part 6: Complete Texts Questions 31 through 34 refer to the following fax.

010-889-880 To: Elizabeth Turner From: Edmund Black Re: Friday‟s meeting Date: January 4, 2006 Liz, I am not ----------- if you have remembered or not, what with the Christmas and New York 31. (A) able (B) sure (C) knowing (D) allowed holidays, but we have a meeting with Dylan Peterson on Friday. As that is only three days --------32. (A) ago (B) before (C) away (D) until I‟d really like to meet up and talk with you beforehand. We really need to look as though we know what we‟re talking about, so I think it would be helpful to meet and make some notes so that we can present a -------------- argument. I think he‟s going to be quite 33. (A) convincing (B) contagious (C) conquering (D) certainty difficult to persuade, so we need to have all the facts well-organized and at our fingertips. I‟m preparing some spreadsheet. Are you --------------- at 10 a.m. tomorrow morning? Give me a call. 34. (A) sure (B) definite (C) free (D) at cost Thanks, Edmund

99

Questions 35 through 38 refer to the following notice.

Today is the first year anniversary of the ------- of the Happy Valley Shopping Centre. In 35. (A) building (B) start (C) opening (D) closure celebration of this fact, stores throughout the Happy Valley complex will be holding special events and big discount sales starting today and ----------- for the next five days. Now is the 36. (A) running (B) completing (C) organizing (D) ran time to check out all of your favorite stores and maybe treat yourself to something nice. Shoes and boots will be half price on Monday. Coats and jackets will be marked ---------37. (A) up (B) down (C) in (D) for 25% on Tuesday. Home furnishings are priced 15% off on Wednesday, the bookstores are offering a 30% discount on novels on Thursday and on Friday, and the food court is ------38. (A) outsourcing (B) supplying (C) serving (D) creating free meals to the first 200 customers! Don‟t miss out, and be sure to tell your friends.

100

Questions 39 through 41 refer to the following notice.

Good personal hygiene is required of all our food service employees. Please refer to the --------- guidelines. 39. (A) under (B) followed (C) following (D) below Hands must be clean at all times, and you are required to wash them with soap and hot water at the beginning of your shift and at--------- once every hour thereafter, after 40. (A) least (B) most (C) all (D) many handling trash, and before returning to work from the restroom or meal breaks. Nails are to be trimmed to a reasonable length. The maximum acceptable length is 1/8 inch longer than your fingertip. Failure to adhere to these guidelines ---------- result in your 41. (A) has to (B) could have (C) might (D) should being asked to leave work for the day, and repeated failure or refusal to adhere WILL result in job loss.

101

Questions 42 through 45 refer to the following advertisement.

Byfords Books Annual Sale Coming! Byfords Books is pleased to ----- its annual discount sale, starting at 10 a.m. on January 42. (A) renounce (B) denounce (C) announce (D) determine 20th. Take a look at just some of the bargains you could ---------43. (A) occur (B) drop (C) pick (D) pick up Bestsellers fiction: Paperbacks- 20% off last year‟s top-selling books Handcover-30% off last year‟s top-selling books. Buy two “Blue Star” books and get a third completely --------------- of charge! 44. (A) lack (B) drop (C) best (D) free

Non-fiction: Biographies – 10% of all books (hardcover and paperback editions) Business – 15% off Buy two “Red Star” books and get a third for half price. Up to 50% off a wide range of titles. Come along and treat yourselves to those books you‟ve been waiting to buy. ** Offers do not ----------- to textbooks or periodicals. ** 45. (A) apply (B) concern (C) offer (D) acknowledge

102

Part 7: Reading comprehension Question 46 through 49 refer to the following notice.

Notice To: Sales Staff FROM: Linda Glass, Sales Manager RE: Monthly Tip Don‟t ask customers, “How much would you like to spend?” or “What is your budget?” These questions give the customers the idea that the price is negotiable, and if they give a low number, then it‟s up to convince them to spend more money, which could be tricky. If the customer does have a budget, and you start to go over it, he or she will let you know anyway. It‟s far more effective to focus on the value of your offering, not its price. If you can convince the customer of the greater value of an item, the price becomes less important.

46. How many times a year does this memo come out? (A) 52 times a year (B) 12 times a year (C) 24 times a year (D) 6 times a year 47. What is the center of a sale, from the salesperson‟s viewpoint? (A) Money (B) Budget (C) Value (D) Negotiations 48. According to the memo, which of the following would be an effective opening sales remark? (A) This is one of the best lines of portable stereos made today. (B) Before we start, let‟s talk about cost. (C) How much were you planning on spending? (D) How does this price fit into your budget? 49. What does Linda Glass want the sales staff to do? (A) Trick the customers (B) Increase monthly sales (C) Emphasize product value (D) Negotiate lower prices

103

Questions 50 through 53 are based on the following advertisement .

Part-time Help Wanted School Bus Driver-Jones Transport Service If you are a good driver who likes children, this is a great opportunity to plan ahead for a terrific part-time job during the second year! Our small buses transport approximately 15 students to and from Steuben County schools within the greater Tri-State Area. At Jones Transport, you are guaranteed a minimum of 20 hours per week at $10.00 per hour after training is complete. We even provide state minimum wage (currently $6.00) during training! We are now accepting applications for our training course. Only five positions will be filled, so hurry and act fast! Parents, bring up to two preschoolers with you on the bus and save on day care expenses while you work! Retirees earn extra spending money and have the summer off to spend it! Live alone? Get out early each morning and enjoy contact with children and their parents. All you need to bring to the job is the ability to relate well to school-aged children, an open, honest and enthusiastic attitude, and a level of comfort behind the wheel of a small school bus. We bring the rest! We are an equal opportunity employer and our training is covered by Veteran‟s Administration Benefits. Call us at 355-1128 Please note that as per state law 45-119640a(g).i4 we cannot employ anyone who has been convicted of drunk driving or any felony. This law applies to convictions within the state as well as within other states. No exceptions will be made to this policy.

50. What is the hourly salary for people while training at Jones Transport? (A) It is $6.00 per hour (B) It is $10.00 per hour (C) People who are training do not get paid (D) It depends on the number of students per bus 51. Why should applications be submitted as soon as possible? (A) The summer training course has already started (B) The school year will start by the end of the month (C) Only a limited number of positions will be filled. (D) Jones Transport buses start running early. 52. Who is NOT specifically encouraged to apply for the job? (A) People who live alone (B) Residents of Steuben County (C) Retired people (D) Parents with young children 53. How many students usually take the bus each day? (A) Five (B) Fifteen (C) Fifty (D) Fifty-five 104

Questions 5 4 through 56 are based on the following advertisement

Ticket-King Do you need tickets for the big event? We’ve got all your ticket needs covered! We sell tickets for sporting events (football, hockey, baseball, even boxing), concerts (from Diana Krall to U2), and the theatre. Special this week only: 10% discount on all concert tickets. That’s right, you can get your pass to the big Tragically Hip concert for just $45* after the discount. Buy yours today! Visit us at www.ticket-king.com * discount price does not include Ticket-King’s additional 70% service charge

54. Where can we buy tickets? (A) In a department store (B) In the newspaper (C) On the internet (D) In the shopping mall 55. For which event can you get a 10% discount? (A) A Michael Jackson concert (B) A hockey game (C) A boxing match (D) a play 56. How much does a ticket for the Tragically Hip concert normally cost? (A) $40 (B) $45 (C) $50 (D) $55

105

Questions 5 7 through 60 are based on the following article.

Many of us wish we were in a better-paid or more interesting job but don‟t know how to start looking for something else. If this describes you, the first thing you need to do is to update your resumé. A resumé is a tool that can be used to obtain a job interview. Along with a cover letter, it is the first impression a prospective employee makes on a potential employer. Therefore, it is important that a resume' provides as much relevant information as possible while remaining brief. A resumé should be no more than two sides of A4 paper. Make it any longer and it won‟t get read. A resumé must be neatly typed, with at least ¾ inch margins on all four sides. Use underlining, capital letters, and asterisks to highlight important information. A resumé should be single-spaced with an extra line of space between blocks of information. Begin a resumé with your name, address, and home and business telephone numbers. Do not include age, marital status, or other personal facts. Next, many resumés list a career goal, followed by a chronological outline of work experience, starting with the most recent job and working backward including a brief description of relevant duties and skills. Finally, include an outline of your educational background from the most recent, backward. Include dates, schools and diplomas awarded. Before you send it, make sure you check your spelling and grammar, or get someone else to go over it if you can.

57. What is the best title for this piece? (A) How to get a job (B) How to prepare a resumé (C) How to find your goal in life (D) How to write a cover letter 58. What is the function of a resumé? (A) To describe your employment and educational background in extensive detail (B) To provide an impressive example of your writing (C) To act as a toll for getting you a job interview (D) To give an employer a good first impression on you. 59. On a resumé, which job should be listed first? (A) Your most recent job (B) Your most relevant job (C) Your first job (D) Your most important job 60. Which of the following should NOT be on your resumé? (A) Your name, telephone, and fax number (B) The year you received your bachelor‟s degree (C) Your nationality and the fact that you are single (D) A description of the tasks you did in your last job 106